AUC Anatomy Block 2

Ace your homework & exams now with Quizwiz!

A 3-day-old infant has a small area of the right iris missing, and a diagnosis of coloboma of the iris is made. Which of the following is the most likely embryologic cause of the coloboma? A. Failure of the retinal/choroid fissure to close B. Abnormal neural crest formation C Abnormal interactions between the optic vesicle and ectoderm D. Posterior chamber cavitation E. Weak adhesion between the inner and outer layers of the optic vesicle

A. A coloboma of the iris is caused by failure of the retinal fi ssure to close during the sixth week

A 36-year-old man is admitted to the emergency department with a painful skin rash on the dorsum of his nose. Physical examination reveals that a herpetic lesion is affecting the dorsum of the nose and the eyeball. Which of the following nerves is most likely to be responsible for transmission of the virus to the eye? A. Nasociliary B. Supratrochlear C. Infraorbital D. Posterior ethmoidal E. Anterior ethmoidal

A. A herpes rash on the dorsum of the nose is known as Hutchinson's sign. This indicates that the virus is located in cell bodies of the ophthalmic division of the trigeminal nerve; This nerve branche into nasociliary, frontal and lacrimal branches. The nasociliary nerve has direct branches th t carry sensory innervation from the eye.

A 3-month-old infant is diagnosed with abnormal face, thymic hypoplasia, cleft palate, hypocalcemia, and a ventricular septal defect. Which of the following genes is defective? A. 22q11 B. SONIC HEDGEHOG C. PAX 2 D. PAX 6 E. 47 XXY

A. Abnormal face, cardiac defects, thymic hypoplasia, cleft palate, and hypocalcemia are characteristics of DiGeorge syndrome. A deletion of the long arm of chromosome 22 (22q11) causes this developmental defect.

A 32-year-old man is admitted to the hospital with nausea, vomiting, and severe headache An MRI examination reveals an acoustic neuroma as shown in Fig. 7-9 . Which of the following nerves is most likely compressed by the tumor? A. Facial B. Oculomotor C. Vagus D. Hypoglossal E. Abducens

A. An acoustic neuroma (vestibular schwannoma or neurolemmoma) is a benign tumor of the vestibulocochlear nerve, which causes compression of VII nerve

A 24-year-old man is admitted to the hospital after a street fight. Radiographic examination reveals an inferior blow-out fracture of the orbit. Which of the following nerves is particularly vulnerable to this type of injury? A. Infraorbital B. Supratrochlear C. Frontal D. Inferior alveolar E. Optic

A. An inferior fracture of the orbit would likely damage the infraorbital nerve. A blow-out fracture often results in a displaced orbital wall, and in this case, the inferior wall.

A 36-year-old woman is admitted to the hospital with severe head injuries after a car crash. During neurologic examination her uvula is deviated to the right. Which nerve is most likely affected to result in this deviation? A. Left vagus B. Right vagus C. Right hypoglossal D. Left glossopharyngeal E. Right glossopharyngeal

A. An injury to the left vagus nerve would cause the uvula to become deviated to the right. If only one side is effectively innervated, contraction of the active muscle will deviate the uvula to the contralateral side of the injury

An unconscious 48-year-old woman is admitted to the hospital. CT scan reveals a tumor in her brain. When she regains consciousness, her right eye is directed laterally and downward, with complete ptosis of her upper eyelid, and her pupil is dilated. Which of the following structures was most likely affected by the tumor to result in these symptoms? A Occulomotor nerve B. Optic nerve C. Facial nerve D. Ciliary ganglion E. Superior cervical ganglion

A. An injury to the oculomotor nerve would cause the eye to point downward and laterally due to the unopposed contractions of the trochlear and abducens nerves

A 22-year-old man is admitted to the emergency department after he was beaten up in a street fight. Radiographic examination reveals that he has suffered a forehead fracture from a blow with a club, resulting in black and swollen eyes ( Fig. 7-6 ). Because the patient is suffering from severe pain, an anesthetic solution is ordered to be injected into his orbit. Which of the following nerves is most likely to be anesthetized? A. Ophthalmic B. Infraorbital C. Anterior ethmoidal D. Frontal E. Optic

A. Anesthetics are injected into the submuscular layer of delicate (areolar) connective tissue, the layer that contains nerves of the eyelid. This space is continuous with the "danger zone" of the scalp.

A 2-month-old male infant is admitted to the hospital after falling from his stroller. During physical examination the infant shows signs of facial nerve injury. What is the most common place for facial nerve injury in an infant? A. At the stylomastoid foramen B. Posterior to the parotid gland C. Anterior to the parotid gland D. Proximal to the stylomastoid foramen E. Mandibular involvement of zygomatic and buccal branches

A. At the point where the facial nerve exits the stylomastoid foramen it is most susceptible to shearing forces. In the absence of a skull fracture whereby the facial nerve can be damaged within the facial canal, the nerve is most commonly injured as it exits the stylomastoid foramen. In infants, in whom the mastoid process has not yet developed, the facial nerve lies unprotected, just beneath the skin.

A 55-year-old male farmer is admitted to the emergency department after falling from the hayloft in his barn. Radiographic examination reveals a small, depressed fracture of the skull vertex and thrombosis of the superior sagittal sinus. A day later the patient loses consciousness. What is the most likely cause of his loss of consciousness? A. Obstruction of CSF resorption B. Obstruction of the cerebral aqueduct (of Sylvius) C. Laceration of the middle meningeal artery D. Fracture of the cribriform plate with CSF rhinorrhea E. Aneurysm of the middle cerebral artery

A. CSF is mostly secreted from the choroid plexuses of the lateral, third, and fourth ventricles of the brain. The CSF enters the subarachnoid space from the fourth ventricle, via the foramina of Luschka and Magendie. The CSF then circulates in the subarachnoid space until it is finally resorbed back into the venous side of the circulation through the arachnoid granulations into the superior sagittal sinus

A 3-month-old infant was admitted to the hospital because of the parents' suspicion that the child was deaf. An MRI examination showed abnormal development of the membranous and bony labyrinths, leading the physician to the diagnosis of congenital deafness . Which of the following conditions can lead to congenital deafness? A. Infection with rubella virus B. Failure of the second pharyngeal arch to form C. Failure of the dorsal portion of first pharyngeal cleft D. Abnormal development of the auricular hillocks E. Failure of the dorsal portion of first pharyngeal cleft and second pharyngeal arch

A. Congenital deafness is due to a maldevelopment of the conducting system of the middle and external ear or neurosensory structures of the inner ear; Rubella infection during a critical time of ear development can lead to a malformed spiral organ (neurosensory hearing loss) or congenital fixation of the stapes, resulting in conducting hearing loss

A 32-year-old woman underwent a thyroidectomy. Two months postoperatively, it was observed that the patient had lost the ability to notice the presence of foreign objects in the laryngeal vestibule. Which of the following nerves was most likely injured? A. Internal laryngeal nerve B. External laryngeal nerve C. Glossopharyngeal nerve D. Hypoglossal nerve E. Recurrent laryngeal nerve

A. Damage to the internal laryngeal nerve would result in a general loss of sensation to the larynx above the vocal cords, leaving the patient with an inability to detect food or foreign objects in the laryngeal vestibule

A 15-year-old male is admitted to the emergency department with severe headache and hydrocephalus. Radiographic examination reveals a craniopharyngioma occupying the sella turcica, primarily involving the suprasellar space. Which of the following is the most likely cause of this tumor? A. Persistence of a small portion of the Rathke pouch B. Abnormal development of pars tuberalis C. Abnormal development of foramina of Monro D. Abnormal development of the alar plates that form the lateral wall of diencephalon E. Abnormal development of diencephalon

A. During embryologic development of the pituitary gland, an outgrowth from the roof of the pharynx (Rathke's pouch) grows cephalad and comprises the anterior lobe (pars distalis) of the pituitary gland.

An 8-year-old male is admitted to the hospital with a drooping right eyelid (ptosis). The initial diagnosis is Horner's syndrome ( Fig. 7-4 ). Which of the following additional signs on the right side would confirm the diagnosis? A. Constricted pupil B. Dry eye C. Exophthalmos D. Pale, blanched face E. Sweaty face

A. Horner's syndrome involves interuption of sympathetic supply to the face This results in ptosis (drooping eyelid) miosis (constricted pupil), and anhydrosis (lack of sweating) of the face

A 2-month-old female infant is hospitalized with hydrocephalus. Radiographic examination reveals cerebrospinal fluid between the compressed brain and overlying bones of the skull. Which of the following conditions will most likely lead to this type of clinical picture? A. Lack of filtration through arachnoid granulations B. Occlusion of cerebral aqueduct (of Sylvius) C. Blockage of the foramina of Luschka D. Congenital absence of the foramen of Magendie E. Closure of the interventricular foramina of Monro

A. If there is CSF between the compressed brain and overlying skull bones, the problem must be a condition of communicating hydrocephalus, with inadequate drainage through the arachnoid granulations into the superior sagittal sinus

A 32 year old woman is undergoing a thyroidectomy. Two months postoperatively the patient suffers from loss of sensation within the larynx from the vocal folds upward to the entrance into the larynx, allowing for aspiration of liquids into the airway. Which of the following nerves is most likely injured? A. Internal laryngeal nerve B External laryngeal nerve C. Glossopharyngeal nerve D. Hypoglossal nerve E. Recurrent laryngeal nerve

A. If there is an injury to the internal laryngeal nerve there is a loss of sensation above the vocal cords.

A 2-month-infant presents with small and flat maxillary, temporal and zygomatic bones. In addition, the patient has anotia and a dermoid tumor in the eyeball. Which of the following conditions is the most likely diagnosis? A. Hemifacial microsomia B. Treacher Collins syndrome C. Robin Sequence D. DiGeorge syndrome E. Velocardiofacial syndrome

A. In hemifacial microsomia the craniofacial anomalies that usually occur involve small and fl at maxillary, temporal, and zygomatic bones. Ear and eye anomalies also occur with this syndrome.

A 1-day old infant presents with a telencephalic vesicle; the eyes are fused, and a single nasal chamber is present in the midline. In addition, the olfactory bulbs and tracts and the corpus callosum are hypoplastic. Which of the following is the most likely diagnosis? A. Holoprosencephaly B. Smith-Lemli-Opitz syndrome C Schizencephaly D. Exencephaly E Meningoencephalocele

A. In holoprosencephaly, loss of midline structures results in malformations of the brain and face

While at summer camp, a 10-year-old boy develops severe pharyngitis and swollen tonsils. Infection may spread from the nasopharynx to the middle ear cavity along the derivative of which embryonic pharyngeal pouch? A First B. Second C. Third D. Fourth E. Sixth

A. Infection can spread from the nasopharynx to the middle ear by way of the auditory tube, which opens to both spaces. The first pharyngeal pouch is responsible for formation both of the auditory tube and middle ear cavity.

A 17-year-old female visits the family dermatologist because of severe facial acne. During physical examination it was found that there was a rather obvious and painful lesion on the side of her nose. The patient was given antibiotics and warned not to press or pick at the large, inflamed swelling. If she were to squeeze, prick, or incise such a lesion in the area between the eye and the upper lip, or between the eye and the side of the nose, the infection could spread to the cavernous sinus. Which of the following pathways of spread of infection would be most typical? A. Nasal venous tributary to angular vein, to superior ophthalmic vein, then to cavernous sinus B. Retromandibular vein to supraorbital vein, then to inferior ophthalmic vein, then to cavernous sinus C Dorsal nasal vein to superior petrosal vein, then inferior ophthalmic vein to cavernous sinus D. Facial vein to maxillary vein, then middle meningeal vein to cavernous sinus E. Transverse facial vein to superficial temporal vein to emissary vein to cavernous sinus

A. Infection in the danger area of the face can lead to cavernous sinus thrombosis because infection spreads from the nasal venous tributary to the angular vein, then on to the superior ophthalmic vein, which passes into the cavernous sinus

A 9-year-old girl is admitted to the emergency department with a painful swelling behind her ear. An MRI examination reveals mastoiditis ( Fig. 7-12 ). Which of the following structures is most likely to be affected by the inflammation? A. Transverse sinus B. Petrous part of the temporal bone C. Middle ear D. Occipital sinus E. Internal carotid artery

A. Mastoiditis is an infection of the air cells within the mastoid process of the temporal bone, often caused by untreated acute otitis media. A known complication of mastoiditis is inflammation of the transverse sinus.

A newborn infant presents with severe brain abnormalities. The calvaria is defective and the brain is protruding from the cranium. A rudimentary brainstem and some functioning neural tissue are present. A diagnosis is made of meroencephaly. Which of the following is the most likely cause of this condition? A. Failure of the rostral neuropore to close in the fourth week B. Cytomegalovirus infection C. Failure of the hypophyseal diverticulum to develop D. Failure of the neural arch to develop E. Abnormal neural crest formation

A. Meroencephaly often results from a failure of the rostral neuropore to close during the fourth week of development. The calvaria is absent, with a resultant extrusion of the brain from the cranium.

A 5-day-old infant is diagnosed with a noncommunicating hydrocephalus. Which of the following is most likely to lead to such a condition? A. Obstruction in the circulation of the cerebrospinal fluid B. Excess production of cerebrospinal fluid C. Increased size of the head D. Disturbances in the resorption of cerebrospinal fluid E. Failure of the neural tube to close

A. Noncommunicating hydrocephalus, also known as obstructive hydrocephalus, is due to an obstruction to fl ow of CSF within the ventricular system.

A 12-year-old boy is admitted to the emergency department with signs of meningitis. To determine the specific type of meningitis, it is necessary to aspirate cerebrospinal fluid with a lumbar puncture for laboratory examination. However, before performing a lumbar puncture, it must be established that the cerebrospinal fluid pressure is not elevated. What condition in the eye would indicate that cerebrospinal fluid pressure is too elevated for a lumbar puncture to be performed? A. Papilledema B. Separation of the pars optica retinae anterior to the ora serrata C. The foveal centralis exhibits hemorrhage from medial retinal branches. D. Obvious opacity of the lens E. Pitting or compression of the optic disc

A. Papilledema is optic disc swelling ("edema of the papilla") that is caused by increased intracranial pressure and increased CSF pressure.

A 34-year-old woman is admitted to the emergency department after her right cheekbone and bony orbit hit the dashboard in an automobile crash. Physical examination reveals that the patient has lost the ability for the affected eye to be directed downward when the pupil is in the adducted position. An MRI examination reveals a torn nerve. What is the most common location at which this nerve will be injured? A. As it pierces the dura of the tentorium cerebelli in the tentorial notch B. At the cavernous sinus C. At the sella turcica D. At the inferior orbital fissure E. At the superior orbital fissure

A. Paralysis of the trochlear nerve results in loss of ability for the affected eye to be directed downward when the pupil is in the adducted position (the primary action of the superior oblique muscle); the delicate nerve is easily torn where it pierces the dura of the tentorium cerebelli in the tentorial notch because the brain and brainstem move forward and backward with the force of impact (a "coup-contrecoup" injury).

A 3-year-old boy is brought to the outpatient clinic with a swelling in the side of his neck. Physical examination reveals a congenital mass of tissue anterior to the superior third of the sternocleidomastoid muscle ( Fig. 7-10 ). The swelling is asymptomatic, nonpainful, and soft. Which of the following is the most likely diagnosis? A. Branchial cleft cyst B. Ruptured sternocleidomastoid muscle C. Lymph node inflammation D. Torticollis E. External carotid artery aneurysm

A. Pharyngeal (branchial) cleft cysts are the most common congenital cause of a neck mass They are epithelial cysts that arise anterior to the superior third of the sternocleidomastoid muscle (1) from a failure of obliteration of the second branchial cleft in embryonic development

A 54-year-old man is admitted to the hospital due to severe headaches. A CT examination reveals an internal carotid artery aneurysm inside the cavernous sinus. Which of the following nerves would be typically affected first? A. Abducens nerve B. Oculomotor nerve C. Ophthalmic nerve D. Maxillary nerve E. Trochlear nerve

A. The abducens nerve would be affected fi rst due to aneurysmal dilation of the internal carotid artery (ICA) because the nerve runs in closest proximity to the artery within the cavernous sinus

A 55-year old woman has undergone facial surgery for the excision of a malignant parotid tumor. A week postoperatively, marked weakness is seen in the musculature of the patient's lower lip. Which of the following nerves was most likely injured during the parotidectomy ? A. Marginal mandibular branch of facial B. Zygomatic branch of facial C. Mandibular division of the trigeminal nerve D. Buccal branch of facial E. Buccal nerve

A. The anterior division of the facial nerve passes through the parotid gland and is therefore at risk during surgery of the parotid gland

A 5-year-old girl is admitted to the hospital with an upper respiratory tract infection. During physical examination her sense of hearing appears to be poor. Her right ear is painful, and upon otoscopic examination a golden brown fluid can be observed through the tympanic membrane. Which is the most likely direct route for the spread of an infection from the upper respiratory tract to the middle ear cavity? A. Pharyngotympanic tube B. Choanae C. Nostrils D. Facial canal E. Internal acoustic meatus

A. The auditory (eustachian or pharyngotympanic) tube is a mucosal-lined tube that provides a direct connection from the nasopharynx to the middle ear cavity.

A 45-year-old woman is admitted to the hospital with severe headache. The patient is diagnosed with hypertension and arrhythmias To reduce the patient's blood pressure, massage is initiated at a pressure point located deep to the anterior border of the sternocleidomastoid muscle at the level of the superior border of the thyroid cartilage. Which of the following structures is targeted by the massage? A. Carotid sinus B Carotid body C Thyroid gland D. Parathyroid gland E. Inferior cervical ganglion

A. The carotid sinus is a baroreceptor that can be targeted for carotid massage to decrease blood pressure. The carotid sinus receptors are sensitive to changes in pressure. For this reason, sustained compression of the carotid sinuses can lead to unconsciousness or death as the heart rate is reflexively reduced.

A 55-year-old woman is admitted to the emergency department with chest angina. ECG examination reveals an acute myocardial infarction. A series of medications is administered to the patient, including sublingual nitroglycerin for reducing her blood pressure. Which of the following structures is most likely to be the route of absorption of this drug? A. Deep lingual vein B. Submandibular duct C. Sublingual duct D. Lingual vein E. Sublingual vein

A. The deep lingual vein is located most superficially on the underside of the tongue. It is therefore the most direct route for absorption of the administered nitroglycerin.

A 56-year-old woman is admitted to the hospital with eye pain. During physical examination the patient complains of excruciating pain when she performs any movement of the eye. An MRI examination reveals that the optic nerve is inflamed What is the most likely explanation? A. The anular tendon (of Zinn) is inflamed. B. The inflammation has affected the nerves innervating the eye muscles. C. The muscles are contracting due to generalized inflammation. D. The nasociliary nerve is affected. E. The ophthalmic artery is constricted

A. The dural covering of the optic nerve is connected to the anular tendon; therefore, when there is an inflammation of the optic nerve, contractions of the recti can evoke severe pain.

A 56-year-old man is diagnosed with an extradural tumor in the posterior cranial fossa. When the patient protruded his tongue during physical examination, the tongue deviated to the right. Which of the following muscles and nerves are most likely injured? A. Right hypoglossal nerve and right genioglossus B. Left hypoglossal nerve and left genioglossus C. Right hyoglossus and left styloglossus D. Right geniohyoid and first cervical nerve E. Contralateral vagus and hypoglossal nerves

A. The hypoglossal nerve innervates the muscles of the tongue and is therefore directly involved in alteration of shape and movement of the tongue A lesion in this nerve would cause deviation of the tongue toward the injured side, which could be observed upon protrusion of the tongue. The genioglossus is the major muscle involved in protrusion of the tongue.

A 16-year-old female volleyball player is admitted to the hospital after being hit in the eye with a ball spiked at the net. Radiographic examination reveals a blow-out fracture of the inferior wall of the orbit. Physical examination also reveals that the pupil of her eye cannot be turned upward. Which of the following muscles is (are) most likely injured? A. Inferior rectus and inferior oblique B. Medial and inferior recti C. Inferior oblique D. Medial rectus, inferior rectus, and inferior oblique E. Inferior rectus

A. The inferior rectus and inferior oblique muscles are entrapped in the crack between the parts of the fractured orbital floor.

A 55-year-old man with severe ear pain visits the ENT doctor. During otoscopic examination the tympanic membrane is ruptured. Which of the following nerves is responsible for the sensory innervation of the inner surface of the tympanic membrane? A. Glossopharyngeal B. Auricular branch of facial C. Auricular branch of vagus D. Great auricular E. Lingual

A. The inner surface of the tympanic membrane is supplied by the glossopharyngeal nerve

A 2-day old infant was born with a cleft palate. The major portion of the palate develops from which of the following embryonic structures? A. Lateral palatine process B. Median palatine process C. Intermaxillary segment D. Median nasal prominences E Frontonasal eminence

A. The largest part of the palate is formed by the secondary palate, which is embryologically derived from the lateral palatine processes

A 35-year old male patient is admitted to the hospital with severe headaches. A CT scan evaluation reveals a tumor in the infratemporal fossa. Physical examination reveals loss of general sensation from the anterior two thirds of his tongue, but taste and salivation are intact. Which of the following nerves is most likely affected by the tumor? A. Lingual proximal to its junction with the chorda tympani B Chorda tympani C. Inferior alveolar D. Lesser petrosal E. Glossopharyngeal

A. The lingual nerve joins the chorda tympani in the infratemporal fossa, and a lesion to the lingual nerve before it joins the chorda tympani would ac count for the loss of general sensation, with no loss to the special sense of taste and saliva production

A 3-day-old male has a noticeably small mandible. A CT scan and physical examinations reveal hypoplasia of the mandible, cleft palate, and defects of the eye and ear. Abnormal development of which of the following pharyngeal arches will most likely produce such symptoms? A. First arch B. Second arch C. Third arch D. Fourth arch E. Sixth arch

A. The listed symptoms are typical of fi rst arch syndrome because the fi rst arch normally gives rise to muscles of mastication, mylohyoid, anterior belly of the digastric, tensor tympani, tensor veli palatini, malleus, and incus

A 55-year-old man is admitted to the emergency department after slipping on wet pavement and falling. Physical examination reveals that the patient has a hematoma that formed in the danger zone of the scalp, spreading to the area of the eyelids. Which of the following layers is regarded as the "danger zone"? A. Loose, areolar layer B. Skin C. Galea aponeurotica D. Pericranium E. Subcutaneous layer

A. The loose areolar connective tissue layer is known as the "danger zone" because hematoma can spread easily from this layer into the skull by means of emissary veins that pass into and through the bones of the skull

A 22-year-old man is admitted to the emergency department with a sinus infection. Radiographic examination reveals posterior ethmoidal cell infection. During physical examination the patient complains of progressive loss of vision. Which of the following structures is most likely affected? A. Ophthalmic artery B. Nasociliary nerve C. Anterior ethmoidal nerve D. Trochlear nerve E. Optic nerve

A. The ophthalmic artery is a branch of the internal carotid artery and provides origin to the ocular and orbital vessels, including the central artery of the retina, which supplies the retina6

A 35-year-old female is under general anesthesia. Prior to laryngeal intubation the rima glottidis is opened by which pair of muscles? A. Posterior cricoarytenoids B. Lateral cricoarytenoids C. Thyroarytenoids D. Transverse arytenoids E. Cricothyroids

A. The posterior cricoarytenoid muscles lie on the superoposterior aspect of the lamina of the cricoid cartilage. When these muscles contract, they cause lateral rotation (abduction) of the vocal processes of the arytenoid cartilages, thereby opening the space between the vocal folds, the rima glottidis

A 38-year-old man is admitted to the hospital with a large mass in his lower anterior neck. Ultrasonic examination reveals a benign tumor of his thyroid gland. Twenty-four hours following a partial thyroidectomy, it was noted that the patient could not abduct the true vocal cords due to a nerve injury during the operation. Which of the following muscles was most likely dennervated? A. Posterior cricoarytenoid B. Lateral cricoarytenoid C. Thyroarytenoid D. Arytenoid E. Cricothyroid

A. The posterior cricoarytenoids are the only muscle of the larynx that abducts the vocal cords

A 55-year-old man is admitted to the emergency department with a complaint of pain when chewing over the previous 3 months. Physical examination reveals the patient suffers from odynophagia and some hoarseness in his speech. Radiographic examination reveals a tumor at the tracheoesophageal groove. Which of the following nerves is most likely affected by the tumor? A. Recurrent laryngeal B. Internal laryngeal C. Vagus D. External laryngeal E. Phrenic

A. The right and left recurrent laryngeal nerves loop around he right subclavian artery and the arch of the aorta, respectively These nerves then travel superiorly in the tracheoesophageal groove to the larynx. Damage to the recurrent laryngeal as a result of surgical intervention or the presence of a tumor in the tracheoesophageal groove would render the patient hoarse.

A 67-year-old female is admitted to the emergency department with a severe swelling on the right side of her neck. An MRI examination reveals an abscess. The abscess is surgically removed from the middle of the posterior cervical triangle on the right side. During recovery the patient notices that her shoulder droops and she can no longer raise her right hand above her head to brush her hair. Which of the following nerves has most likely been iatrogenically injured? A. Accessory B. Ansa cervicalis C.Facial D. Hypoglossal E. Suprascapular

A. The spinal accessory nerve passes across the posterior triangle of the neck and innervates both the trapezius muscle and the sternocleidomastoid muscle for the respective side of the body

A 64-year-old man is admitted to the hospital in an unconscious condition. A CT scan examination reveals that the patient has suffered a cerebral vascular accident (stroke), with a small hematoma produced by the superior cerebellar artery. Which of the following nerves will most likely be affected by the hematoma? A. Trochlear nerve B. Abducens nerve C. Facial nerve D. Vestibulocochlear nerve E. Glossopharyngeal nerve

A. The superior cerebellar artery arises near the termination of the basilar artery, passes immediately below the oculomotor nerve, and eventually winds around the cerebral peduncle, close to the trochlear nerve, as it continues on toward the upper surface of the cerebellum where it will divide into branches that anastomose with the inferior cerebellar arteries.

A 36-year-old woman is admitted to the hospital with severe head injuries after a car crash. During neurologic examination it is noted that her uvula is deviated to the right. Which of the following muscles is paralyzed? A. Left levator veli palatini B. Left tensor veli palatini C. Right levator veli palatini D. Right tensor veli palatini E. Right tensor veli palatini and left levator veli palatini

A. The uvula would move toward the intact right side. This is because the intact levator veli palatini would be unopposed by the opposite, paralyzed left levator veli palatini.

A 3-day-old male infant has a noticeably small mandible. A CT scan and physical examinations reveal hypoplasia of the mandible, underdevelopment of the bones of the face, downward-slanting palpebral fissures, defects of the lower eyelids, and deformed external ears. Abnormal development of which of the pharyngeal arches will most likely produce such symptoms? A. First arch B. Second arch C. Third arch D. Fourth arch E. Sixth arch

A. The first pharyngeal arch, which is often associated with the mandible, is responsible for development of Meckel's cartilage, malleus, incus, and mandible. Additionally, it is innervated by the trigeminal nerve, specifically the mandibular division that innervates the muscles of mastication

A 62-year-old man visits the outpatient clinic complaining of spontaneous lacrimation during eating. Which of the following nerves has developed a lesion to cause this condition? A. Facial nerve proximal to the geniculate ganglion B. Greater petrosal nerve C. Lesser petrosal nerve D. Lacrimal nerve E. Chorda tympani

A. There is a lesion of the facial nerve proximal to the geniculate ganglion. At the geniculate ganglion the greater petrosal nerve branches from the facial nerve and ultimately runs to the pterygopalatine ganglion where preganglionic fi bers synapse on postganglionic neurons that innervate the lacrimal gland. There is a disruption of the facial nerve proximal to this branch that allows the greater petrosal nerve to be stimulated by factors that would normally stimulate the submandibular and sublingual glands. These glands are innervated via the chorda tympani that comes off the facial nerve distal to the geniculate ganglion

A 3-year-old boy is admitted to the hospital because of a soft, anterior, midline cervical mass. When the patient is asked to protrude his tongue, the mass in the neck is observed to move upward. Which of the following is the most likely diagnosis? A. A thyroglossal duct cyst B. Defect in sixth pharyngeal arch C. A branchial cyst D. Cystic fistula of the third pharyngeal arch E. Defect in first pharyngeal arch

A. Thyroglossal duct cysts occur due to retention of a remnant of the thyroglossal duct along the path followed by the descending thyroid gland during development.

A 3-month-old male infant is brought to the hospital by his parents because of white patches in his eyes. An ophthalmoscopic examination shows a congenital cataract. Which of the following conditions can cause a congenital cataract? A. Infection with rubella virus B. Choroid fissure fails to close C. Persistent hyaloid artery D. Toxoplasmosis infection E. Cytomegalovirus infection

A. With congenital cataracts, the lens appears opaque and grayish white and blindness will result. Infection by teratogenic agents such as rubella virus (German measles) can cause congenital cataracts.

A 33-year-old woman is unconscious when she is admitted to the hospital after she fell, hitting her head. The physician in the emergency department performs a pupillary light reflex test. The integrity of which of the following nerves is being checked? A. Optic and facial B. Optic and occulomotor C. Maxillary and facial D. Ophthalmic and occulomotor E. Ophthalmic and facial

B The optic and oculomotor nerves are responsible for the sensory and motor portions, respectively, of the pupillary light reflex

A 14-year-old female has been suffering from quinsy in the right side of her oropharynx. In surgical removal of the pathologic tissue, or for incision and drainage of the area, which of the following arteries will be at greatest risk? A. Lingual B. A branch of facial C. Superior laryngeal artery D. Ascending pharyngeal artery E. Descending palatine artery

B. A branch of the facial artery would be of primary concern because its branches supply the oropharynx and it is the primary source of arterial supply to the palatine tonsil

A 34-year-old woman is admitted to the emergency department with a painful eye. Physical examination reveals a lump in the lower eyelid that consists of debris ( Fig. 7-13 ). A diagnosis of a chalazion is made Which of the following structures is (are) most likely blocked by the chalazion? A. Lacrimal ducts B. Tarsal glands C. Sclera D. Pupil E. Nasolacrimal duct

B. A chalazion is caused by an obstructed tarsal gland of the eyelid. Swellings of the lacrimal gland usually present on the upper lateral eyelid and are not indicative of a chalazion. A chalazion is not an infection within the eye, so this excludes sclera and pupil from bei g the correct answers. The nasolacrimal duct runs from the medially located lacrimal sacs to the inferior meatus of the nose and would be unaffected in the case of a chalazion

A 22-year-old male is admitted to the hospital after he was hit in the right eye with a frozen fish, thrown playfully by a friend while they were passing through the seafood section of the market. During physical examination considerable swelling and discoloration of the eyelids was observed. In addition, the patient could not turn his pupil laterally from forward gaze, indicating probable muscle entrapment. Which of the following bones was most likely fractured? A. Orbital plate of the frontal bone B. Lamina papyracea of the ethmoid bone C. Orbital plate of the maxilla D. Cribriform plate of the ethmoid bone E. Greater wing of the sphenoid bone

B. A fracture of the lamina papyracea of the ethmoid bone is likely to entrap the medial rectus muscle, causing an inability to gaze laterally

A 57-year-old man is admitted to the emergency department with dizziness and severe headaches. A CT scan evaluation reveals a tumor in the superior orbital fissure. Upon physical examination the patient's eye ball is fixed in an abducted position, slightly depressed, and the pupil is dilated. In addition, the superior palpebrae are ptotic. When the patient is asked to move the pupil toward the nose, the pupil rotates medially. Consensual corneal reflexes are normal. Which of the following nerves is most likely affected? A. Trochlear nerve B. Oculomotor nerve C. Abducens nerve and sympathetic nerve plexus accompanying the ophthalmic artery D. Ophthalmic nerve and short ciliary nerve E. Superior division of oculomotor nerve and the nasociliary nerve

B. A lesion of the oculomotor nerve will cause the eye to remain in a "down and out" position. This is due to the actions of the unopposed lateral rectus (supplied by the abducens nerve) and the superior oblique (supplied by the trochlear nerve).

A 67-year-old man visits the outpatient clinic with complaints of deteriorating vision. A form of glaucoma is diagnosed in which the aqueous humor does not drain properly into the scleral venous sinus at the iridoscleral angle of the eyeball. The aqueous fluid is secreted by the epithelium of the ciliary body directly into which of the following spaces? A. Iridoscleral angle B. Posterior chamber C. Pupil D. Vitreous body E. Lacrimal sac

B. Aqueous humor is secreted by the ciliary body into the posterior chamber of the eye. The humor flows through the pupil into the anterior chamber and then is filtered by a trabecular meshwork, then drained by the canal of Schlemm

A 27-year-old woman is admitted to the emergency department after she was thrown from a motor scooter. Radiographic evaluation reveals a type I Lefort fracture and comminuted fracture of the mandible and TMJ. Despite reconstructive surgery, the patient develops hyperacusis (sensitivity to loud sounds) due to facial nerve paralysis. Which of the following muscles is most likely paralyzed? A. Posterior belly of digastric B. Stapedius C. Tensor tympani D. Stylohyoid E. Cricothyroid

B. Both the stapedius and tensor tympani normally function to dampen movements of the middle ear ossicles, thereby muting sound and preventing hyperacusis.

A 32-year-old man is admitted to the emergency department with visual problems. Radiographic examination reveals a tumor of the adenohypophysis (anterior pituitary gland). Physical examination reveals a loss of the lateral halves of the fields of vision of both eyes (bitemporal hemianopia or "tunnel vision"). Which of the following structures was most likely compressed by the tumor? A. Optic nerve B. Optic chiasm C. Optic tract D. Oculomotor E. Abducens nerve

B. Compression of the optic chiasm can cause bitemporal hemianopia due to compression of nerve fibers coming from the nasal hemiretinas of both eyes. The optic chiasm is located in very close proximity above the pituitary gland

A 34-year-old man is admitted to the emergency department after falling off his motorbike, suffering an injury to his head. The patient has multiple lacerations in the skin over the frontal bone. Which of the following veins could most likely provide a pathway of transmission of infection from the veins of the scalp to the underlying dural venous sinuses? A. Supratrochlear vein B. Diploic veins C. Anterior cerebral veins D. Superior sagittal sinus E. Supraorbital vein

B. Diploic veins are responsible for communication between the veins of the scalp and the venous sinuses of the brain. Diploic veins are situated within the layers of bone of the skull and connect the emissary veins of the scalp to the venous sinuses located between two layers of dura

A 6-month-old infant is admitted to the emergency department with hydrocephalus. Upon physical examination a spina bifida cystica is noted. Radiographic examination reveals a caudal displacement of the cerebellar structures through the foramen magnum. Which of the following is the most likely diagnosis? A. Arnold-Chiari malformation B. Holoprosencephaly C. Smith-Lemli-Opitz syndrome D. Schizencephaly E. Exencephaly

B. Holoprosencephaly is caused by failure of the prosencephalon to properly divide into two cerebral hemispheres

A 55-year-old woman is admitted to the emergency department with ear pain, ringing in her ear (tinnitus), dizziness, and vertigo. Radiographic examination reveals indications of Ménière s disease. Which of the following structures is most likely affected by the edema that is associated with Ménière's disease? A. Middle ear B. Endolymphatic sac C. Semicircular canals D. Cochlea E. Helicotrema

B. Hydrops (edema) results from accumulation of excessive fluid in the endolymphatic sac. Labyrinthine hydrops or endolymphatic hydrops is known as Ménière disease.

A 45-year-old man came to the outpatient clinic after stumbling and hitting his head on a table in a restaurant. During the neurologic examination photographs were taken of the patient's eyes as shown in left eye was rotated medially . Which of the following nerves to the left eye was most likely injured? A. Trochlear B. Abducens C. Oculomotor D. Optic E. Oculomotor and abducens

B. If the left abducens nerve is injured, there will be a loss of function of the left lateral rectus muscle so the patient will be unable to abduct his left eye.

A 54-year-old woman is admitted to the emergency department after experiencing sudden problems with her vision for the preceding 5 days. Radiographic examination reveals that an aneurysm of one of the arteries at the base of the brain is compressing the optic chiasm. Which of the following arteries will most likely be involved? A. Middle cerebral B. Anterior communicating C. Anterior cerebral D. Superior cerebellar E. Posterior superior cerebellar

B. The anterior communicating artery, the portion of the arterial circle (of Willis), is directly superior to the optic chiasm, and an aneurysm of this artery would likely compress the chiasm, as in this patient.

The arachnoid villi allow cerebrospinal fluid to pass between which two of the following spaces? A. Choroid plexus and subdural space B. Subarachnoid space and superior sagittal sinus C. Subdural space and cavernous sinus D. Superior sagittal sinus and jugular vein E. Epidural and subdural space

B. The arachnoid villi are extensions of the arachnoid mater into the superior sagittal sin s. The villi allow for proper drainage of the CSF into the venous bloodstream from the subarachnoid space in which the CSF circulates

A 56-year-old woman is admitted to the hospital with rheumatoid arthritis of her temporomandibular joint (TMJ) and severe ear pain. Which of the following nerves is most likely responsible for conducting the pain sensation? A. Facial B. Auriculotemporal C. Lesser petrosal D. Vestibulocochlear E. Chorda tympani

B. The auriculotemporal nerve is a posterior branch of the mandibular division of the trigeminal nerve. It encircles the middle meningeal artery and courses medially to the TMJ and then ascends up near the auricle

A 6-year-old boy is admitted to the hospital with high fever and pain over the parotid gland ( Fig. 7-7 ). A diagnosis of parotiditis (mumps) is established, and the boy is sent back home. Which of the following nerves is responsible for painful sensations from the region of the parotid gland? A. Facial B. Auriculotemporal C. Lesser petrosal D. Lingual E. Chorda tympani

B. The auriculotemporal nerve leads into the parotid gland, and its compression in mumps can be associated with severe pain. The compressive effects are due in large part to the continuity of the facial capsule of the parotid gland with the tough layer of superficial investing fascia of the neck, a layer that is almost non-distensible. When the gland swells, sensory fibers for pain are triggered rapidly, and can be referred to the ear. None of the other nerves listed supply the parotid gland.

An unconscious 57 year-old man is transported to the emergency department after falling from a tree. A CT scan evaluation reveals a fracture of the cribriform plate ( Fig 7-5 ). Which of the following conditions will most likely be present during the physical examination? A Entrapment of the eyeball B. Anosmia C. Hyperacusis D. Tinnitus E. Deafness

B. The axons of olfactory nerves run directly through the cribriform plate to synapse in the olfactory bulb Damage to this plate can damage the nerve axons, causing anosmia (loss of the sense of smell).

A 46-year-old woman is admitted to the hospital with a large mass in her lower anterior neck. Ultrasonic examination reveals a benign tumor of her thyroid gland. During the procedure to remove the tumor the superior thyroid artery is identified and used as a landmark in order not to damage its small companion nerve. Which of the following nerves is most likely to accompany the superior thyroid artery? A. Cervical sympathetic trunk B. External branch of the superior laryngeal C. Inferior root of the ansa cervicalis D. Internal branch of the superior laryngeal E. Recurrent laryngeal

B. The external branch of the superior laryngeal nerve courses together with the superior thyroid artery for much of its route

A 34-year-old male complains of hyperacusis sensitivity to loud sounds). Injury to which of the following cranial nerves is responsible? A. Hypoglossal B. Facial C. Accessory D. Vagus E. Glossopharyngeal

B. The facial nerve innervates the stapedius muscle, which is responsible for limiting movement of the stapes, thereby reducing the intensity of the sound entering the inner ear

A 10-year-old girl is admitted to the hospital with tonsillitis. A tonsillectomy is performed and the tonsils are removed On physical examination one week later the patient has absence of the gag reflex on the left when the posterior part of the tongue is depressed. The sensory portion of which of the following nerves was most likely injured? A. Facial B. Glossopharyngeal C. Mandibular D Maxillary E. Hypoglossal

B. The gag reflex is composed of both an afferent and an efferent limb. These reflexes are mediated by the glossopharyngeal and vagus nerves, respectively.

Cardiac pain is referred in some cases to the mandible and the region of the TMJ. Cutaneous sensation over the angle of the mandible is normally supplied by which of the following nerves? A Cervical branch of facial B. Great auricular nerve C. Mandibular branch of trigeminal nerve D. Mandibular branch of facial nerve E. Transverse cervical nerve

B. The great auricular nerve is derived from the ventral rami of the second and third cervical nerves and supplies the skin over the angle of the mandible up to the level of the TMJ

A 55-year-old woman is admitted to the hospital with severe hypertension. Laboratory examination reveals hypertension (190/110 mm Hg) and hypercholesterolemia (250 mg/dl). During physical examination the patient complains of headaches and dizziness. Radiographic examination reveals 90% occlusion of both common carotid arteries A carotid endarterectomy is performed and large atherosclerotic plaques are removed. During a postoperative physical examination on the right side, it was noted that her tongue deviated toward the right when she was asked to stick it out. Which of the following nerves was most likely injured during the procedure? A. Right glossopharyngeal B. Right hypoglossal C. Left hypoglossal D. Left lingual E. Left vagus

B. The hypoglossal nerve provides motor innervation to the muscles of the tongue, with the exception of the palatoglossus. Injury to the hypoglossal nerve would result in deviation of the tongue toward the affected side when the tongue is protruded

A 38-year-old patient is admitted to the dental clinic with acute dental pain. The attending dentist found penetrating dental caries (tooth decay) affecting one of the mandibular molar teeth. Which of the following nerves would the dentist need to anesthetize to remove the caries in that tooth? A. Lingual B. Inferior alveolar C. Buccal D. Mental E. Mylohyoid

B. The inferior alveolar branch of the mandibular division of the trigeminal nerve provides sensory innervation to the mandibular teeth and would require anesthesia to abolish painful sensation

A 27-year-old man is admitted to the hospital after a middleweight boxing match. During physical examination the strength and symmetry of strength in opening the jaws are tested. Which of the following muscles is the most important in jaw protrusion and depressing the mandible? A. Anterior portion of temporalis B. Lateral pterygoid C. Medial pterygoid D. Masseter E. Platysma

B. The lateral pterygoid muscle is a muscle of mastication innervated by the lateral pterygoid nerve of the mandibular division of the trigeminal nerve.

A 35-year-old man is admitted to the hospital with severe pain in the area of his right submandibular gland. Radiographic examination reveals a tumor of the gland. An incision is made and the submandibular gland and its duct are removed. Which of the following nerves is most commonly injured in this type of procedure? A. Buccal B. Lingual C. Inferior alveolar D. Nerve to mylohyoid E. Glossopharyngeal

B. The lingual nerve initially courses directly underneath the mucosa of the floor of the mouth and superficial to the submandibular gland, specifically the submandibular duct. This nerve is therefore at risk for ligation, division, or trauma during excision of the gland and duct.

A 1-year-old infant is admitted to the hospital with fever. Radiographic examination reveals a sinus infection. Which of the following sinuses is present at this age? A. Frontal sinus B. Maxillary sinus C. Sphenoid sinus D. Middle ethmoidal air cells E. Posterior ethmoidal air cells

B. The maxillary sinus arises late in fetal development and is the only sinus present at birth. The frontal and sphenoid sinuses often develop at approximately 2 years of age from the anterior ethmoid air cells and the posterior ethmoid air cells, respectively

A 54-year-old man is admitted to the hospital with severe pain in his nasal cavity. Radiographic examination reveals a carcinoma in his nasal cavity. In which of the following locations would the carcinoma block the hiatus of the maxillary sinus? A. Inferior meatus B. Middle meatus C. Superior meatus D. Nasopharynx E. Sphenoethmoidal recess

B. The maxillary sinus drains via the middle meatus, specifically via the semilunar hiatus. The middle meatus and semilunar hiatus are located under the middle nasal concha

A 45-year-old man is admitted to the hospital with breathing problems. During physical examination the patient shows signs of airway obstruction. A CT scan examination reveals a nasal polyp obstructing the airway ( Fig. 7-14 ). Drainage from which of the following structures is also obstructed? A. Sphenoid sinus B. Maxillary sinus C. Ethmoidal sinus D. Frontal sinus E. Nasolacrimal duct

B. The nasal polyp also involved the maxillary sinus, located immediately laterally to the nasal cavity.

A 31-year-old mother visits the pediatric outpatient clinic with her 6-month-old baby complaining that her baby is not developing quickly and has no teeth. Which of the following teeth are expected to appear first? A. Superior medial incisors at 8 to 10 months of age B. Inferior medial incisors at 6 to 8 months of age C. Superior lateral incisors at 8 to 10 months of age D. Inferior lateral incisors at 12 to 14 months of age E. First molar at 6 to 8 months of age

B. The order of tooth eruption is a follows: inferior medial incisors (6 to 8 months), superior medial incisors (8 to 10 months), first molar (6 to 8 months), superior lateral incisors (8 to 10 months), and finally inferior lateral incisors (12 to 14 months).

An 11-year-old boy with swollen palatine tonsils is examined by an otolaryngologist. Which of the following arteries supplies most of the blood to these tonsils and must be protected when its tonsillar branch is divided? A. Ascending pharyngeal B. Facial C. Lingual D. Descending palatine E. Superior thyroid

B. The palatine tonsils are highly vascular and are primarily supplied by the tonsillar branch of the facial artery; therefore, care is taken to preserve this artery while performing a tonsillectomy

A 62-year old man is admitted to the hospital with blurred vision. Physical examination reveals a long history of gradual loss of his visual field. The intraocular pressure is high, and a diagnosis of glaucoma is made. Which of the following spaces first receives the aqueous humor secreted by the epithelium of the ciliary body? A. Anterior chamber B. Posterior chamber C. Pupil D. Vitreous E. Lacrimal sac

B. The posterior chamber receives ciliary body secretions fi rst. The ciliary body produces aqueous humor and is located in the posterior chamber

A 54-year-old male is to undergo bilateral thyroidectomy. During this procedure there is the possibility of bilateral paralysis of muscles that can open the airway. If a particular nerve is injured bilaterally, there is significant risk of asphyxiation postoperatively unless the patient is intubated or the airway is opened surgically. Which of the following muscle pairs opens the airway? A. Cricothyroids B. Posterior cricoarytenoids C. Arytenoideus D. Thyroarytenoids E. Lateral cricoarytenoids

B. The posterior cricoarytenoid muscle is the only abductor of the larynx that opens the rima glottidis and rotates the arytenoid cartilages laterally

A 56-year-old woman has just undergone a complete thyroidectomy. After she recovers from the anesthesia a hoarseness of her voice is noted that persists for 3 weeks. Subsequent examination shows a permanently adducted vocal fold on the right side. Surgical trauma to the innervation of which of the following muscles is most likely to be responsible for the position of the right vocal fold? A. Aryepiglottic B. Posterior cricoarytenoid C. Thyroarytenoid D. Transverse arytenoids E. Vocalis

B. The recurrent laryngeal nerve is often at risk of being damaged during a thyroidectomy. Patients who have a transected or damaged recurrent laryngeal will often present with a characteristic hoarseness following surgery.

A 55-year-old man is admitted to the emergency department with fever of 4 days' duration. Radiographic examination reveals the presence of an infection that is spreading from the retropharyngeal space to the posterior mediastinum. Between which of the following fascial layers are the infections most likely located? A. Between alar and prevertebral B. Between alar and pretracheal C. Between pretracheal and prevertebral D. Between buccopharyngeal and alar E. Between buccopharyngeal and prevertebral

B. The retropharyngeal space extends from the inferior aspect of the skull to the posterior mediastinum. An infection or abscess in this space could thus travel toward the posterior mediastinum. The retropharyngeal space is enclosed between the visceral fascia covering the posterior wall of the pharynx and the alar layer of the prevertebral fascia

A 47-year-old woman is admitted to the hospital with signs of cavernous sinus thrombosis. Radiographic examination reveals a pituitary tumor involving the cavernous sinus, confirming the initial diagnosis ( Fig 7-3 ). During physical examination it is suspected that the right abducens nerve of the patient has been damaged by the tumor. In which direction will the physician most likely ask the patient to turn her right eye to confirm the abducens nerve damage, assuming she is unable to perform this task? A. Inward B. Outward C. Downward D. Down and out E. Down and in

B. The right abducens nerve innervates the right lateral rectus, which mediates outward movement (abduction) of the right eye

A 1-day-old infant was born with the vault of the skull undeveloped, leaving the malformed brain exposed. A diagnosis of exencephaly is made. What is the embryologic cause of this condition? A. Toxoplasmosis infection B. Failure of closure of the cephalic part of the neural tube C. Ossification defect in the bones of the skull D. Caudal displacement of cerebellar structures E. Maternal alcohol abuse

B. The rostral neuropore closes during the fourth week of development. If this does not occur, the forebrain primordium is abnormal and the calvaria or vault fails to develop

A 55-year-old man is admitted to the hospital after an injury sustained at work in a factory. He presents with severe scalp lacerations, which were sutured. After three days the wound is inflamed, swollen, and painful. Between which tissue layers is the infection most likely located? A. The periosteum and bone B. The aponeurosis and the periosteum C. The dense connective tissue and the aponeurosis D. The dense connective tissue and the skin E. The dermis and the epidermis

B. The scalp is divided into five layers: skin, dense connective tissue, aponeurosis, loose connective tissue, and periosteum. Typically, infections will be located in the loose connective tissue because of the ease with which infectious agents spread via the many veins located in this region

A 68-year-old woman is suffering from excruciating, sudden bouts of pain over the area of her midface. Laboratory studies indicate that she has tic douloureux (trigeminal neuralgia). Which ganglion is the location of the neural cell bodies of the nerve mediating the pain? A. Geniculate B. Trigeminal (semilunar or Gasserian) C. Inferior glosopharyngeal D. Otic E. Pterygopalatine

B. The semilunar ganglion, also known as the trigeminal or Gasserian ganglion, is the location of the sensory neuron cell bodies of the trigeminal nerve

A 48-year-old man presents with a constricted right pupil that does not react to light. His left pupil and vision in both eyes are normal. These findings are most likely due to a lesion involving which of the following right-sided structures? A. Oculomotor nerve B. Superior cervical ganglion C. Nervus intermedius D. Edinger-Westphal nucleus E. Trigeminal (semilunar, Gasserian) ganglion

B. The superior cervical ganglion provides sympathetic innervation o the face and neck regions

A 35-year-old woman is hospitalized due to cavernous sinus thrombosis resulting from an infection on the face. Which of the following is the most direct route for spread of infection from the face to the cavernous sinus? A. Pterygoid venous plexus B. Superior ophthalmic vein C. Frontal venous plexus D. Basilar venous plexus E. Parietal emissary vein

B. The superior ophthalmic vein drains directly into the cavernous sinus. The danger area of the face is located in the triangular region from the lateral angle of the eye to the middle of the upper lip, near the nose, and is drained by the facial vein

A patient with enlarged cervical lymph nodes has a malignant tumor of the cecum. Which of the following lymph nodes of the neck is most frequently associated with malignant tumors of the gastrointestinal tract? A. Left inferior deep cervical B. Left supraclavicular C. Right inferior deep cervical D. Right supraclavicular E. Jugulodigastric

B. The supraclavicular lymph node on the left side is associated with the thoracic duct. The thoracic duct receives lymph from below the diaphragm, including the gastrointestinal tract. Malignant cells that travel up the thoracic duct are known to involve the left supraclavicular lymph node

An unconscious 54-year-old female is admitted to the hospital. A CT scan reveals a tumor in her brain, producing a tentorial herniation. When she regains consciousness, her right eye is directed laterally and downward, with complete ptosis of her upper eyelid and pupillary dilation. Which of the following lobes of the brain is affected by the tumor? A. Parietal B. Temporal C. Occipital D. Frontal E. Parietal and temporal

B. The tentorial/uncal herniation described in this case is most likely to occur as a result of a temporal lobe tumor. The uncus is part of the temporal lobe, and when enlarged, it will be compressed against the foramen magnum. This results in the symptoms manifested by damage to the nearby oculomotor nerve. The uncus is not a part of the other named lobes.

A 34-year-old swimmer presents to your office with an external ear canal infection (otitis externa). The patient coughs during inspection of the external auditory meatus with a speculum. The cough results from the irritation of which nerve that innervates an area of the external auditory meatus? A. Vestibulocochlear B. Vagus C. Trigeminal D. Facial E. Accessory

B. The vagus nerve innervates a part of the external auditory meatus and, when stimulated, can trigger a cough refl ex in about 20% of people

An 11-year-old boy is examined by an ENT doctor for his swollen palatine tonsils. The palatine tonsils are located between the anterior and posterior tonsillar pillars. Which of the following muscles form these pillars? A. Levator veli palatini and tensor veli palatini B. Palatoglossus and palatopharyngeus C. Styloglossus and stylopharyngeus D. Palatopharyngeus and salpingopharyngeus E. Superior and middle pharyngeal constrictors

B. These pillars are formed by the palatoglossal arch, anteriorly, and the palatopharyngeal arch posteriorly.

A 55-year-old man is admitted to the hospital with a complaint of severe headaches. A lumbar puncture reveals traces of blood in the cerebrospinal fluid. Which of the following conditions has most likely occurred in this patient? A. Fracture of the pterion with vascular injury B. A ruptured "berry" aneurysm C. Leakage of branches of the middle meningeal vein within the temporal bone D. A tear of the cerebral vein at the entrance to the superior sagittal sinus E. Occlusion of the internal carotid artery by a clot generated in the left atrium

B. When a berry aneurysm ruptures, the blood fl ows into the subarachnoid space and therefore mixes with CSF; thus, blood would be present in the CSF when a lumbar puncture is performed.

A 45-year-old man is admitted to the emergency department with a red, painful eye. During physical examination it is noted that the conjunctiva of the affected eye is infected Which of the following lymph node groups would be first involved if the infection spread? A. Submandibular B Parotid C. Jugulodigastric D. Submental E. Preauricular

B?

A 36-year-old female racquetball player is admitted to the hospital after being struck in the orbital region. Radiographic examination reveals a blow-out fracture of the medial wall of the orbit. Physical examination also reveals that the pupil of the affected eye cannot be turned laterally. Which of the following muscles is most likely injured or trapped? A. Lateral rectus B. Medial and inferior recti C. Medial rectus D. Medial rectus and superior oblique E. Inferior rectus

C. A blow-out fracture of the medial wall of the orbit would likely render the medial rectus nonfunctional by entrapment of the muscle between the edges of the cracked medial wall

A 7-year-old boy was suffering from a severe infection of the middle ear (otitis media), which spread to the mastoid air cells (mastoiditis). Surgery was required but resulted in the following: right corner of the mouth drooping, unable to close his right eye, food collection in his right oral vestibule. Which nerve was injured? A. Glossopharyngeal B. Vagus C. Facial D. Maxillary division of the trigeminal nerve E. Mandibular division of the trigeminal nerve

C. A lesion of the facial nerve is likely to lead to the symptoms described (drooping mouth, unable to close right eye, and food collection in the oral vestibule because the muscles of facial expression are paralyzed

A 5-day-old infant was born with a normal Apgar score. One month later the external acoustic meatus was atretic Which of the following conditions was the most likely cause of this defect? A. Otic pit did not form. B. Development of the first pharyngeal pouch was affected. C. Meat l plug did not canalize. D. Auricular hillocks did not develop. E. The tubotympanic recess degenerated.

C. A normal Apgar score indicates that the child appeared normal and healthy at birth, based on skin color, heart rate, refl exes, muscle tone, and breathing. An atretic external acoustic canal occurs due to failure of the meatal plug to canalize

A 58-year-old man is admitted to the emergency department with progressive unilateral hearing loss and ringing in the affected ear (tinnitus) of 4 months' duration. Radiographic examination reveals a tumor at the cerebellopontine angle. Which of the following nerves is most likely affected? A. Vagus B. Hypoglossal C. Vestibulocochlear D. Glossopharyngeal E. Trigeminal

C. A tumor at the cerebellopontine angle, such as an acoustic schwannoma, is most likely to affect first the vestibulocochlear nerve and then the facial nerve

A 54-year-old man is admitted to the hospital with severe headaches. A CT scan reveals a tumor in his brain occupying a portion of the anterior cranial fossa. Which of the following is responsible for the sensation of pain from headache in this case? A Meningeal branches of the maxillary nerve B. Meningeal branches of the mandibular nerve C. Meningeal branches of the ethmoidal nerve D. Tentorial nerve E. C2 and C3 fibers

C. A tumor involving the meningeal branches of the ethmoidal nerves that originate from the ophthalmic division of the trigeminal nerve is likely to cause pain from pressure and nerve injury in the anterior cranial fossa.

A 5-week-old male infant is born without a thymus or inferior parathyroid glands. Which of the following pharyngeal arches is most likely involved? A. First B. Second C. Third D. Fourth E. Fifth

C. Absence of the thymus and inferior parathyroid glands would be due to defective development of the third pharyngeal pouch, their normal site of origin

A 22-year-old woman is admitted o the emergency department unconscious after falling over the handlebars of her bicycle. An emergency tracheotomy is performed to insert a tracheotomy tube. What is the most common tracheal cartilage level at which a tracheotomy incision is performed? A First to second B. Second to third C. Third to fourth D. Fourth to fifth E. Fifth to sixth

C. An incision at the level of the third and fourth tracheal cartilages usually results in the fewest complications during a tracheostomy

A 4-year-old boy suffering from ankyloglossia is brought to the speech therapist. The examining physician recommends that the child be admitted for operation by a pediatric surgeon. Which of the following surgical procedures would be most appropriate for this condition? A. Removal of pterygomandibular raphe B. Resection of the pterygoid hamulus bilaterally C. Cutting the lingual frenulum D. Repair of the palate E. Removal of the central segment of the hyoid bone

C. Ankyloglossia (tongue-tie) is characterized by a lingual frenulum that extends all the way to the tip of the tongue. This condition can cause problems with speech, feeding, and oral hygiene as a result of the low range of motion of the tongue

A 6-year-old male child, whose medical history includes a complicated delivery, has a permanently tilted head posture, with the right ear near the right shoulder and the face turned upward and to the left. Which of the following muscles was most likely damaged during birth? A. Anterior scalene B. Omohyoid C. Sternocleidomastoid D. Trapezius E. Platysma

C. Because of its size and vulnerable position during birth, the sternocleidomastoid muscle is injured more often than other muscles of the head and neck during birth

A 43 year-old man is diagnosed with laryngeal carcinoma. A surgical procedure is performed and the tumor is successfully removed from the larynx. The right ansa cervialis is anastomosed with the right recurrent laryngeal nerve in order to reinnervate the muscles of the larynx and restore phonation. Which of the following muscles will most likely be paralyzed after this operation? A. Sternocleidomastoid B. Platysma C. Sternohyoid D. Trapezius E. Cricothyroid

C. Because of the surgical division of the ansa cervicalis, the sternohyoid muscle will most likely be paralyzed following this tumor resection; The ansa cervicalis innervates the strap muscles, including the sternohyoid, sternothyroid, and omohyoid muscles

A 3-month-old male infant has a lump in his neck. A biopsy of the lump shows it to be thymic tissue. Based on embryonic origin, which of the following additional structures is most likely to have an ectopic location? A. Jugulodigastric lymph node B. Lingual tonsil C. Parathyroid gland D. Submandibular gland E. Thyroid gland

C. Both the inferior parathyroid glands and the thymus are derived from the third pharyngeal pouch.

A 34-year-old woman is admitted to the hospital with a large mass in her lower anterior neck. Ultrasonic examination reveals a benign tumor of her thyroid gland. Twenty-four hours following a partial thyroidectomy, it was noted that the patient frequently aspirates fluid into her lungs. Upon examination it was determined that the area of the piriform recess above the vocal fold of the larynx was anesthetized. Which of the following nerves was most likely iatrogenically injured? A. External branch of the superior pharyngeal B. Hypoglossal C. Internal branch of the superior laryngeal D. Lingual E. Recurrent laryngeal

C. During removal of the tumor, the internal branch of the superior laryngeal nerve was injured. Injury to this nerve results in loss of sensation above the vocal cords, at the entrance to the larynx, and loss of taste on the epiglottis

During the routine ophthalmologic exam, the globe, the retina, and the cornea of each eye are tested Which of the following nerves must be functioning properly if the patient is to be able to turn the eye laterally (abduction) without difficulty and without upward or downward deviation? A. Superior division of oculomotor, ophthalmic nerve, abducens nerve B. Trochlear nerve, abducens nerve, nasociliary nerve C. Inferior division of oculomotor, trochlear, abducens D. Oculomotor and ophthalmic nerves E. Superior division of oculomotor, trochlear, and abducens nerves

C. For proper movements of the eye to occur, all cranial nerves of the extraocular eye muscles are required (oculomotor, trochlear, and abducens nerves) The inferior division of the oculomotor innervates the inferior rectus, the medial rectus, and the inferior oblique.

A 20-year old man is admitted o the emergency department with a stab wound in the superior region of his neck. A radiographic examination reveals that the wound has not affected any major structures. Physical examination reveals that the patient has lost sensation from the skin over the angle of the jaw. Which of the following nerves is most likely injured? A. Supraclavicular B. Transverse cervical C Great auricular D Greater occipital E Lesser occipital

C. Four nerves participate in providing cutaneous supply to the neck. The nerves are the supraclavicular, great auricular, transverse cervical, and the lesser occipital. The area over the angle of the jaw is innervated by the great auricular nerve

A 45-year-old woman is admitted to the hospital with a swelling on the side of her face of 2 months' duration. Radiographic examination reveals a parotid gland tumor. An operative procedure is performed in which the tumor is removed from the parotid gland. Three months postoperatively the patient complains that her face sweats profusely when she tastes or smells food, and a diagnosis is made of Frey syndrome (gustatory sweating). Which of the following nerves was most likely injured during the procedure? A. Buccal B. Inferior alveolar C. Auriculotemporal D. Facial E. Lingual

C. Frey syndrome occurs following damage to parasympathetic axons in the auriculotemporal nerve; As the peripheral nerves make new connections, aberrant connections can be formed between the auriculotemporal nerve and other glands (not usually innervated by the auriculotemporal nerve).

A 65-year-old man is admitted to the emergency department after his head hit the dashboard in an automobile collision. Radiographic and physical examinations reveal that the inferior alveolar nerve is injured at its origin. Which of the following muscles would most likely be paralyzed as a result? A. Geniohyoid B. Hyoglossus C. Mylohyoid D. Stylohyoid E. Palatoglossus

C. Just before it passes into the mandible to supply the lower teeth and chin, the inferior alveolar nerve gives rise to the mylohyoid nerve, a motor nerve supplying the mylohyoid and anterior belly of the digastric

A 40-year-old woman presents with severe headaches and dizziness. An MRI reveals a brain tumor, and a biopsy confirms it as a melanoma. She dies 2 months later. Pigmented lesions are not seen on her skin or scalp at the time of diagnosis or during postmortem examination. Which of the following is the most likely source of the malignant melanoma cells? A. Superior sagittal sinus B. Sphenoidal sinus C. Retina of the eye D. Pituitary gland E. Thymus

C. Melanocytes in the pigmented layer of the retina are a potential source of malignant melanoma cells The tumor spreads hematogenously directly to the brain and has a very poor prognosis. None of the other listed structures contains melanocytes.

An 8 year-old boy had an extensive mastoidectomy due to an infection that did not respond to antibiotics. Postoperatively he had Bell's palsy (facial paralysis), and one of the features was that saliva tended to accumulate in the vestibule of his oral cavity and dribble from the corner of his mouth. Which of the following muscles was paralyzed to allow this condition to occur? A. Zygomoaticus major B. Orbicularis oculi C. Buccinator D. Levator palpebrae superioris E. Orbicularis oris

C. Normally the tonus of the buccinator muscle prevents the accumulation of saliva and foodstuffs in the oral vestibule. Although a lesion of the facial nerve would paralyze the other muscles listed, the buccinator is the most important muscle of the cheek.

A 5-day-old infant male has an abnormally large head. A CT scan examination reveals enlarged lateral and third ventricles but a normal-size fourth ventricle. Stenosis of the cerebral aqueduct (of Sylvius) is suspected. Which of the following conditions will be characteristic of these symptoms? A. Nonobstructive hydrocephalus B. Anencephaly C. Obstructive hydrocephalus D. Meroanencephaly E. Holoprosencephaly

C. Obstructive hydrocephalus, in this case resulting from obstruction of the cerebral aqueduct, refers to a condition in which fl ow of cerebrospinal fluid (CSF) is obstructed within the ventricular system

A 12-year-old girl is admitted to the emergency department with a middle ear infection. Physical examination reveals a long history of chronic middle ear infections that have produced a lesion in the tympanic plexus in the middle ear cavity. Since the preganglionic parasympathetic fibers that pass through the plexus have been lost, which of the following conditions will be detectable during physical examination? A. Diminished mucus in the nasal cavity B. Diminished mucus on the soft palate C. Diminished saliva production by the parotid gland D. Diminished saliva production by the submandibular and sublingual glands E. Diminished tear production by the lacrimal gland

C. Parasympathetic innervation of the parotid gland is provided by axons carried by the glossopharyngeal nerve that emerge from the tympanic plexus of the middle ear as the lesser petrosal nerve.

A 67-year-old man visits the outpatient clinic with hearing problems. During physical examination a Rinne test for hearing is performed by placing a tuning fork on his head to test for bone conduction. Upon what specific point should the tuning fork be placed to test conduction? A. Temporal bone B. Frontal bone C. Mastoid process D. External occipital protuberance E. Vertex of the head

C. The Rinne test is often employed during physical examination to determine possible conduction hearing loss. A tuning fork is struck and placed on the mastoid process

A 22-year-old woman is admitted to the hospital with an injury to her eye. The corneal reflex is tested and found to be present. Which of the following nerves is responsible for the afferent limb of this reflex? A. Frontal B. Lacrimal C. Nasociliary D. Oculomotor E. Optic

C. The afferent/sensory limb of the corneal (blink) refl ex is carried by the nasociliary nerve. It is a branch of the ophthalmic division of the trigeminal nerve.

A 68-year-old woman visits the outpatient clinic with a complaint of chronic dizziness and headaches. Cranial and cervical angiography ( Fig. 7-11 ) reveals an occluded vessel. Which of the following vessels is most likely occluded? A. External carotid B. Internal carotid C. Common carotid D. Vertebral E. Superior thyroid

C. The angiograph provided clearly shows that the radiopaque medium injected into the patient did not completely fill the common carotid artery

The arterial circle (of Willis) contributes greatly to cerebral arterial circulation when one primary artery becomes occluded by atherosclerotic disease. Which of the following vessels does not contribute to the circle? A. Anterior communicating artery B. Posterior communicating artery C. Middle cerebral artery D. Internal carotid artery E. Posterior cerebral artery

C. The arterial circle (of Willis) receives its blood supply from the internal carotid and vertebral arteries; The middle cerebral artery is the lateral continuation of the internal carotid artery.

A 32-year-old man is admitted to the emergency department unconscious after a severe car crash. During an emergency cricothyroidostomy an artery is accidentally injured. Two days later the patient shows signs of aspiration pneumonia. Which of the following arteries was most likely injured? A. Superior thyroid B. Inferior thyroid C. Cricothyroid D. Superior laryngeal E. Suprahyoid

C. The cricothyroid artery is a small branch of the superior thyroid artery. It anastomoses with the cricothyroid artery of the opposite side at the upper end of the median cricothyroid ligament, a common site for establishing an emergency airway

A 3-month-old male is under observation in the pediatric clinic. The patient has no thymus, congenital parathyroidism, and thyroid hypoplasia. Abnormal development of which of the following pharyngeal pouches or arches will most likely produce these defects? A. First and second B. Second and third C. Third and fourth D. Fourth E. Fourth and sixth

C. The defect is likely in the development of third and fourth pharyngeal pouches because the superior parathyroid glands are derived from the fourth pouch

A 3-year-old girl ruptured her eardrum when she inserted a pencil into her external ear canal. She was urgently admitted to the emergency department. Physical examination revealed pain in her ear and a few drops of blood in the external auditory meatus. There was the concern that there might possibly have been an injury to the nerve that principally innervates the external surface of the tympanic membrane. Which of the following tests is most likely to be performed during physical examination to check for injury to this nerve? A. Check the taste in the anterior two thirds of the tongue. B. Check the sensation to the pharynx and palate. C. Check if there is paraesthesia at the TMJ. D. Check for sensation in the larynx. E. Check for sensation in the nasal cavity

C. The external surface of the tympanic membrane is innervated primarily by the auriculotemporal nerve, a branch of the mandibular division of the trigeminal nerve. Damage to this nerve would additionally result in painful movements of the TMJ because this joint receives innervation from the same nerve

A 55-year-old woman visits the outpatient clinic complaining of loss of sensation in the posterior third of her tongue. Radiographic examination reveals Eagle's syndrome, in which the styloid process and stylohyoid ligament are elongated and calcified. Which of the following nerves is most likely affected by Eagle's syndrome in this patient? A. Vagus B. Facial C. Glossopharyngeal D. Hypoglossal E. Vestibulocochlear

C. The glossopharyngeal nerve enters the posterior oropharynx by coursing between the stylohyoid ligament and the stylopharyngeus muscle Calcification of the stylohyoid ligament can readily affect this nerve by irritation or compression

A 43-year-old man is admitted to the emergency department with a fracture of the base of his skull. A thorough physical examination reveals that a number of structures have been injured, possibly including the right greater petrosal nerve. Which of the following conditions needs to be identified during physical examination to confirm the diagnosis of greater petrosal nerve injury? A. Partial dryness of the mouth due to lack of salivary secretions from the submandibular and sublingual glands B. Partial dryness of the mouth due to lack of salivary secretions from the parotid gland C. Dryness of the right cornea due to lack of lacrimal secretion D. Loss of taste sensation from the right anterior two thirds of the tongue E. Loss of general sensation from the right anterior two thirds of the tongue

C. The greater petrosal nerve carries parasympathetic fibers that are involved in the innervation of the lacrimal gland, as well as the mucosal glands of the nose, palate, and pharynx

A 55-year-old woman is diagnosed with a tumor at the base of the skull, resulting in a decrease in tear production. Which of the following nerves is most likely injured? A. Chorda tympani B. Deep petrosal C. Greater petrosal D. Lesser petrosal E. Nasociliary

C. The greater petrosal nerve, a parasympathetic branch of the facial nerve, provides innervation to the lacrimal gland of the orbit

A 45-year-old man was suffering from trigeminal neuralgia (tic douloureux). The pain was so severe that the patient had considered suicide as a way to escape the pain. Even light, gentle stimuli to the skin between the lower eyelid and the upper lip resulted in severe, agonizing pain. It was decided to lesion the nerve branch involved by injection of alcohol into the nerve. To reach the nerve, the needle will most likely need to be inserted through which of the following openings? A. Foramen ovale B. Foramen spinosum C. Infraorbital foramen D. Mandibular foramen E. Foramen magnum

C. The infraorbital branch of the maxillary division of the trigeminal nerve exits the front of the skull below the orbit through the infraorbital foramen.

An 11-year-old boy visits the outpatient clinic with a history of recurrent infections of his tonsils. Which of the following lymph nodes is most likely to first become visibly enlarged during tonsillitis? A. Submandibular B. Parotid C. Jugulodigastric D. Submental E. Preauricular

C. The jugulodigastric node, also known as the tonsillar lymph node, receives drainage from the tonsils, tongue, and pharynx. It is often enlarged during tonsillitis.

A 5-year-old boy fell from a tree and was admitted to the emergency department unconscious. When an emergency tracheostomy was performed, profuse dark venous bleeding suddenly occurred from the midline incision over the trachea Which of the following vessels was most likely accidentally cut? A. Superior thyroid vein B Inferior thyroid vein C. Left brachiocephalic vein D. Middle thyroid vein E. Jugular arch connecting the anterior jugular veins

C. The left brachiocephalic vein is the most likely vein punctured in the procedure because it extends across the trachea from the left side of the body, joining the right brachiocephalic vein to form the superior vena cava, which is located just to the right of the midline

A 24-year-old male had a third molar (wisdom tooth) extracted from his lower jaw. This resulted in the loss of general sense and taste sensation from the anterior two thirds of the tongue. This loss was most likely due to injury of which of the following nerves? A. Auriculotemporal B. Chorda tympani C. Lingual D. Mental E. Inferior alveolar

C. The lingual nerve is the most likely nerve damaged because there is loss both of taste and general sensory supply to the anterior two thirds of the tongue, which is innervated by the lingual nerve, which at this point has been joined by the chorda tympani.

A 24-year-old man is admitted to the hospital after a street fight. Radiographic examination reveals an inferior (blow-out) fracture of the orbit. Orbital structures would most likely be found inferiorly in which of the following spaces? A. Ethmoidal sinus B. Frontal sinus C. Maxillary sinus D. Nasal cavity E. Sphenoidal sinus

C. The maxillary sinus is located inferior to the orbit.

A 35-year-old woman is admitted to the emergency department after a violent automobile crash. The patient's upper airway is obstructed with blood and mucus, and a midline tracheotomy inferior to the thyroid isthmus is performed. Which of the following vessels are most likely to be present at the site of incision and will need to be cauterized? A. Middle thyroid vein and inferior thyroid artery B. Inferior thyroid artery and inferior thyroid vein C. Inferior thyroid vein and thyroidea ima artery D. Cricothyroid artery and inferior thyroid vein E. Left brachiocephalic vein and inferior thyroid artery

C. The most likely structures one would encounter while performing a midline incision below the isthmus of the thyroid gland would be the inferior thyroid vein and the thyroidea ima artery.

A 3-month-old male infant is admitted to the hospital because he cries continuously During physical examination it is observed that he infant has a dry right eye. Upon the basis of imaging studies, the neuroophthalmologist diagnoses a lesion at the neural cell bodies of the preganglionic axons of the pterygopalatine ganglion. Which of the following structures contains the neural cell bodies of the preganglionic axons? A. Superior cervical ganglion B. Edinger-Westphal nucleus C. Superior salivatory nucleus D. Inferior salivatory nucleus E. Nucleus ambiguus

C. The neural cell bodies whose axons synapse in the pterygopalatine ganglion are located in the superior salivatory nucleus, which is in the pons; this nucleus provides the GVE fibers of the facial nerve for lacrimal and salivary secretion

A 65-year-old man is admitted to the emergency department after an episode of a transient ischemic attack. Radiographic examination reveals an aneurysm in the region between the posterior cerebral artery and superior cerebellar artery. Which of the following nerves will most likely be compressed from the aneurysm? A. Trochlear B. Abducens C Oculomotor D. Vagus E Optic

C. The oculomotor nerve passes between the posterior cerebral artery (PCA) and the superior cerebellar artery near the junction of the midbrain and pons.

A 17-year-old woman is admitted to the hospital with signs of cavernous sinus thrombosis, as revealed by radiographic and physical examinations. Thrombophlebitis in the "danger area" of the face can spread to the cavernous sinus and involve the ophthalmic branch of the trigeminal nerve. Which of the following symptoms will most likely be present during physical examination? A. Pain in the hard palate B. Anesthesia of the upper lip C. Pain from the eyeball D. Pain over the lower eyelid E. Tingling sensation over the buccal region of the face

C. The ophthalmic branch of the trigeminal nerve supplies sensory innervation to the eyeball, leading to p in upon damage

A 32-year-old female patient asks you what is the soft, thin ridge of tissue that she can feel running forward across the masseter muscle toward her upper lip. You reassure her that is perfectly normal. Which of the following is the most likely structure she is feeling? A. Facial artery B. Maxillary artery C. Parotid duct D. Marginal mandibular branch of facial nerve E. Facial vein

C. The parotid duct, also known as the Stensen's duct, crosses the masseter muscle transversely and extends to the oral cavity.

A 45-year-old man with a complaint of ear pain and difficulty hearing is diagnosed with tonsillitis. Otoscopic examination reveals fluid in the middle ea cavity. Hypertrophy of which of the following structures would be most likely to compromise the drainage of the auditory tube? A. Lingual tonsil B. Palatine tonsil C. Pharyngeal tonsil D. Superior constrictor muscle E. Uvula

C. The pharyngeal tonsil is situated in a slit-like space, the pharyngeal recess, in the nasopharynx behind the opening of the auditory (eustachian) tube, and a pharyngeal tonsil in this location can lead to blockage of the drainage of the auditory tube.

A 29-year-old woman underwent a thyroidectomy. Postoperatively, the patient presented with hoarseness. Which of the following nerves was most likely injured during the operation? A. Internal laryngeal B. External laryngeal C. Recurrent laryngeal D. Superior laryngeal E. Glossopharyngeal

C. The recurrent laryngeal nerve supplies most of the motor innervation to the larynx and sensation below the true vocal folds

A 31-year-old female is admitted to the hospital after an automobile collision. A CT scan examination reveals a large hematoma inferior to the right jugular foramen. Physical examination reveals right pupillary constriction (miosis) and anhydrosis (loss of sweating) of the face. Which of the following ganglia is most likely affected by the hematoma? A. Submandibular B. Trigeminal (semilunar or Gasserian) C. Superior cervical D. Geniculate E. Ciliary

C. The superior cervical ganglion (SCG), which is the uppermost part of the sympathetic chain, supplies sympathetic innervation to the head and neck.

A 48-year old male patient complains of diplopia (double vision). On neurologic examination he is unable to adduct his left eye and lacks a corneal reflex on the left side. Where is he most likely location of the lesion resulting in the symptoms? A. Inferior orbital fissure B. Optic canal C. Superior orbital fissure D. Foramen rotundum E. Foramen ovale

C. The superior orbital fissure is the opening that allows the passage of the oculomotor nerve and the trochlear nerve; the lacrimal, frontal, and nasociliary branches of ophthalmic division of the trigeminal nerve; the abducens nerve; the superior and inferior divisions of the ophthalmic vein; and the sympathetic fibers from the cavernous plexus

A 54-year-old man was admitted to the emergency department after he was struck by an automobile. Radiographic examination revealed a fracture through the crista galli of the anterior cranial fossa, resulting in slow, local bleeding. Which of the following is the most likely source of bleeding? A. Middle meningeal artery B. The great cerebral vein of Galen C. Superior sagittal sinus D. Straight venous dural sinus E. Superior ophthalmic vein

C. The superior sagittal sinus would most likely be the source of the bleeding because it attaches anteriorly to the crista galli and because of the slow nature of the bleed.

A 45-year-old female is admitted to the hospital with severe headaches, dizziness, and vomiting. Radiographic examination reveals an intracranial tumor. Upon physical examination the patient has dryness of the nasal and paranasal sinuses, loss of lacrimation,and loss of taste from the anterior two thirds of the tongue. Which of the following structures is most likely involved with the tumor? A. Auriculotemporal nerve B. Lesser petrosal nerve C. Facial nerve D. Inferior salivatory nucleus E. Pterygopalatine ganglion

C. The superior salivatory nucleus is the autonomic nucleus for the facial nerve; Parasympathetic fibers carried by the greater petrosal branch of the facial nerve are responsible for supply of the lacrimal gland and sinuses, via the pterygopalatine ganglion

A 73-year-old male patient visits the outpatient clinic with a complaint of progressive, painless loss of vision. Radiographic examination reveals thrombophlebitis of the cavernous sinus. Through which of the following structures must a thrombus pass to cause the symptoms of this patient? A. Subarachnoid space B. Central artery of the retina C. Central vein of the retina D. Optic chiasm E. Ciliary ganglion

C. The thrombus may pass through the central vein of the retina to reach the cavernous sinus. The patient would suffer blindness because the central vein is the only vein draining the retina and if it is occluded, blindness will ensue.

A 59-year-old painter fell from the scaffolding and was admitted to the emergency department in an unconscious condition. An emergency tracheostomy is performed and brisk arterial bleeding suddenly occurs from the midline incision over the trachea. Which of the following vessels was most likely cut accidentally? A. Inferior thyroid branch of thyrocervical trunk B. Cricothyroid branch of the superior thyroid artery C. Thyroidea ima artery D. Middle thyroid vein E. Jugular arch connecting the anterior jugular veins

C. The thyroidea ima artery supplies the thyroid gland and ascends in the front of the trachea; therefore, it would be easily injured in an emergency tracheostomy with a midline incision over the trachea

A 63-year-old man with hearing loss in his left ear complains of a loss of taste and drooling from the left side of his mouth. A CT scan shows a tumor compressing the nerve entering the skull through which of the following openings? A. Foramen ovale B. Foramen rotundum C. Internal acoustic meatus D. Jugular foramen E. Superior orbital fissure

C. The tumor is compressing the facial nerve, which runs through the internal acoustic meatus along with the vestibulocochlear nerve

A 1-day-old infant presents with meningohydroencephalocele. Which of the following bones is most commonly affected? A. Squamous part of temporal bone B. Petrous part of temporal bone C. Squamous part of occipital bone D. Sphenoid bone E. Ethmoid bone

C. Usually, deficits of the cranium involve the squamous part of the occipital bone and, in some cases, the posterior aspect of the foramen magnum

Cleft lip, with or without cleft palate occurs about once in 1000 births. Which of the following is considered to be the most important causative factor in the production of this anomaly? A. Riboflavin deficiency B. Infectious disease C. Mutant genes D. Cortisone administration during pregnancy E. Irradiation

C. Whereas all forms of clefts are considered to have a multifactorial etiology, cleft lip in particular seems to have a strong genetic factor

A young couple hiking in a wilderness area discovered the body of a man apparently in his 20s. He appeared to have been dead a few days, but animal predation was minimal. A postmortem examination was performed by the county medical examiner, and no evidence of penetrating wounds (bullet, lacerations, e c.) was found. A plain radiograph showed a fractured hyoid bone, but the calvaria and other bones appeared to be intact. Which of the following is the most likely cause of death? A. Myocardial infarction (heart attack) B. A fall from a height that resulted in fatal internal bleeding C. Subdural hematoma D. Strangulation E. Ingestion of a poisonous substance

D. A fractured hyoid bone is evidence of strangulation. A fall from a height and subdural hematoma would likely be accompanied by fractured bones. Whereas myocardial infarction or poison remain possibilities, the medical examiner would have a high index of suspicion for strangulation because of the fractured hyoid bone.

A 1-day-old infant who exhibits absence of the ocular lens is admitted to the pediatric intensive care unit. Laboratory examination reveals a mutation in the PAX6 gene. Which of the following conditions is the most likely diagnosis? A. Cyclopia B Coloboma C. Anophthalmia D. Aphakia and aniridia E. Microphthalmia

D. A mutation of the PAX6 gene usually results in congenital aphakia (absence of lens) and aniridia (absence of iris).

A 21-year-old male baseball player is brought to the emergency department after feeling severe dizziness. During physical examination the patient demonstrates lack of equilibrium and memory impairment. A 3-cm wound is noted in his scalp from an injury suffered in a game several weeks earlier. A lumbar puncture does not reveal blood in the cerebrospinal fluid. Which of the following is the most likely diagnosis? A. The middle meningeal artery was torn, resulting in epidural hematoma. B. There is a fracture in the pterion with injury to the adjacent vasculature C. The injury resulted in the bursting of a preexisting aneurysm of the anterior communicating artery of the cerebral circle. D. A cerebral vein is torn. E. The cavernous sinus has a thrombus

D. A torn cerebral vein often results in a relatively slow-bleeding subdural hematoma. Such a hematoma can be involved in gradual compression of the brain, resulting in confusion, dizziness, clumsiness, and memory loss. There would be no sign of blood in the CSF because the bleeding is into the subdural space, not the subarachnoid space

A 34-year-old man is admitted to the hospital with severe headaches, dizziness, and vomiting. Imaging studies reveal a tumor at the hypoglossal canal. Which of the following muscles will most likely be affected? A. Geniohyoid B. Mylohyoid C. Palatoglossus D. Genioglossus E. Thyrohyoid

D. A tumor at the hypoglossal canal would compress the hypoglossal nerve and affect the genioglossus, a muscle it supplies

A 16-year-old boy is admitted to the hospital with fever, confused mental state, and drowsiness. During physical examination it is noted that the boy suffers from severe acne. Radiologic examination reveals cavernous sinus thrombosis. Which of the following routes of entry to the cavernous sinus would most likely be responsible for the infection and thrombosis? A. Carotid artery B. Mastoid emissary vein C. Middle meningeal artery D. Ophthalmic vein E. Parietal emissary vein

D. Cavernous sinus thrombosis can often result from squeezing pimples or other infectious processes located around the danger area of the face, which includes the area of the face directly surrounding the nose. This physical pressure has the potential to move infectious agents from the pimple into the ophthalmic vein, which then carries into the cavernous sinus

A 54-year-old man is admitted to the emergency department with a fracture at the frontozygomatic suture. During physical examination the eyelid of the patient exhibits multiple lacerations and the sclera contains small fragments from his broken glasses. What site would be preferable for needle insertion to anesthetize the orbital contents and then the area of the eyelid injury? A. Into the sclera in the limbic region and also into the infraorbital foramen B. Into the lacrimal fossa and also beneath the lateral bulbar conjunctiva C. Into the supraorbital foramen and also into the lacrimal caruncle D. Through the upper eyelid deeply toward the orbital apex and also between the orbital septum and the palpebral musculature laterally E. Directly posteriorly through the anulus tendineus and superior orbital fissure

D. It is necessary to anesthetize the conjunctival covering of the sclera, which is supplied by the nasociliary branch of the ophthalmic nerve. To do that, the needle should be placed through the upper eyelid deeply toward the orbital apex to infi ltrate the nasociliary nerve, and also between the orbital septum and the palpebral musculature laterally to anesthetize lateral sensory supply from the lacrimal nerve and (perhaps) twigs from the maxillary nerve

A 21-year-old man was brought to the emergency department because of severe epistaxis (nosebleed) from the nasal septum. This area, knows as Kiesselbach's (or Little's) area, involves mostly anastomoses between which of the following arteries? A. Ascending palatine and ascending pharyngeal B. Posterior superior alveolar and accessory meningeal C. Lateral branches of posterior ethmoidal and middle meningeal D. Septal branches of the sphenopalatine and superior labial E. Descending palatine and tonsillar branches of the pharyngeal

D. Kiesselbach (also called Little) plexus is an anastomosis of four arteries on the anterior nasal septum. The four arteries are the anterior ethmoidal artery, sphenopalatine artery, superior labial artery, and greater palatine artery

A 55-year-old male has a complaint of left-sided maxillary tooth pain. A dental examination reveals no abnormalities of his teeth. During physical examination tapping on his right maxilla elicits sharp pain on the right side of his face. The patient reports that he has no allergies. Which of the following conditions will be the most likely diagnosis? A. Sphenoid sinusitis B. Anterior ethmoidal sinusitis C. Posterior ethmoidal sinusitis D. Maxillary sinusitis E. Frontal sinusitis

D. Maxillary sinusitis is an infection of the max illary sinus, which is located in the body of the maxillary bone

A 63-year-old man had his prostate gland tumor removed 2 years before his present admission to the hospital, complaining of various neurologic problems, including headache. Radiographic examination reveals that the cancer has spread from the pelvis to the posterior cranial fossa by way of the internal vertebral venous plexus (of Batson). During physical examination the patient's right shoulder droops noticeably lower than the left, he exhibits considerable weakness in turning his head to the left, and his tongue points to the right when he attempts to protrude it directly from his mouth. There are no other significant findings. Which of the following nerves are most likely affected? A. Right vagus, right accessory, and right hypoglossal nerves B. Left accessory, right glossopharyngeal, right vagus, and left hypoglossal nerves C. Left hypoglossal, right trigeminal, and left glossopharyngeal nerves D. Right accessory and right hypoglossal nerves E. Left facial, left accessory, right accessory, and vagus nerves

D. Paralysis of the right accessory and hypo glossal nerves. Drooping of the right shoulder occurs as a result of paralysis of the trapezius as a result of injury to the right accessory nerve, which supplies that muscle. Loss of the right accessory nerve would also result in weakness in turning the head to the left, a function of the right sternocleidomastoid muscle which is supplied by this nerve. The tongue deviation to the right is due to the unopposed activity of the left tongue muscles since the right hypoglossal nerve is affected

A 59-year-old man is admitted to the emergency department with acute pain on his mandible. An MRI examination reveals an acute inflammation of the TMJ due to arthritis. Which of the following muscles will most likely be affected by the inflammatory process of the TMJ? A. Temporalis B. Medial pterygoid C. Masseter D. Lateral pterygoid E. Buccinator

D. Part of the lateral pterygoid muscle has its insertion on the articular disk within the TMJ and would be most affected by the inflammation of this joint

A newborn infant is finally delivered with forceps after a difficult delivery. Upon physical examination of the newborn a cephalohematoma is noted from rupture of small periosteal arteries. Between which of the following layers of tissue does the blood accumulate? A. Between skin and dense connective tissue layer B. Between loose connective tissue layer and galea aponeurotica C. Between galea aponeurotica and pericranium D. Between pericranium and calvaria E. At the subcutaneous layer

D. Rupture of the periosteal arteries resulting in a cephalohematoma is defi ned as a collection of blood underneath the periosteum. On the head, it is located between the pericranium (periosteum of the skull) and the calvaria (skull)

A 65-year-old male is admitted to the hospital three weeks after a "small bump of his head" according to his narrative. He suffered the accidental bump from a low-hanging branch while driving his tractor through the apple orchard during harvesting season. During physical examination the patient displays mental confusion and poor physical coordination. Radiographic examination reveals leakage from a cerebral vein over the right cerebral hemisphere. From what type of bleeding is the patient most likely suffering? A. Subarachnoid bleeding B. Epidural bleeding C. Intracerebral bleeding into the brain parenchyma D. Subdural bleeding E. Bleeding into the cerebral ventricular system

D. Subdural bleeding usually results from tears in veins that cross the subdural space, between the dura and the arachnoid. This bleeding may cause a gradual increase in intracranial pressure and may result in leakage of venous blood over the right cerebral hemisphere with a variable rate of progression

A 32-year-old woman is admitted to the hospital after losing consciousness and collapsing in the middle of the street. A neurologic examination reveals absence of the accommodation reflex of her right eye. Which of the following is most likely involved in the pathology in this patient? A. Superior salivatory nucleus B. Superior cervical ganglion C. Nervus intermedius D. Edinger-Westphal nucleus E. Trigeminal ganglion

D. The accommodation reflex is performed by constriction of the pupil when trying to focus on a near object. This function is controlled by the parasympathetic nerve fibers carried in the oculomotor nerve from the Edinger-Westphal nucleus of the midbrain that synapse in the ciliary ganglion

A 50-year-old woman complained of pain over her chin and lower lip. A few days later small vesicles appeared over the same area and soon began erupting. She was diagnosed with a dermatomal herpes zoster inflammation (shingles). Which of the following nerves was most likely responsible for the transmission of the virus in this case? A. Auriculotemporal B. Buccal C. Lesser petrosal D. Mental E. Infraorbital

D. The chin and lower lip area are supplied by the mental nerve, a branch of the inferior alveolar nerve, which in turn is a branch of the mandibular division of the trigeminal nerve

An 8-year-old boy was suffering from a severe infection of the right middle ear. Within the course of a week, the infection had spread to the mastoid antrum and the mastoid air cells. The organisms did not respond to antibiotics, so the surgeon decided to perform a radical mastoid operation. Following the operation, it was noticed that the boy's face was distorted. The mouth was drawn upward to the left and he was unable to close his right eye. Saliva tended to accumulate in his right cheek and dribble from the corner of his mouth. What structure was most likely damaged during the operation? A. Mandibular nerve B. Parotid duct C. Vagus nerve D. Facial nerve E. Glossopharyngeal nerve

D. The facial nerve exits the skull via the stylomastoid foramen, just anterior to the mastoid process. A lesion of the facial nerve is likely to cause the symptoms described as a result of paralysis of the facial muscles

A 17-year-old woman is admitted to the hospital with tonsillitis. A tonsillectomy is performed and the patient complains postoperatively of ear pain. Which of the following nerves was most likely injured during the surgical procedure? A. Auriculotemporal B. Lesser petrosal C. Vagus D. Glossopharyngeal E. Chorda tympani

D. The glossopharyngeal nerve mediates general somatic sensation from the pharynx, the auditory tube, and from the middle ear. Painful sensations from the pharynx, including the auditory tube, can be referred to the ear by this nerve, as in this case of tonsillectomy.

You wake one morning to discover that your alarm has not worked and you are running late. Desperate to get to your biochemistry lecture in time, yet unbearably hungry, you quickly throw some bread in the toaster as you get ready. Despite the toast burning a little, you eat it quickly as you rush out the door. The burnt parts of the toast scratch the roof of your mouth, leaving you with a stinging sensation there. What nerve is collecting this sensation from the hard palate? A. Posterior superior alveolar nerve B. Inferior alveolar nerve C. Lingual nerve D. Greater palatine nerve E. Lesser palatine nerve

D. The greater palatine nerve is responsible for innervation of the hard palate, or the hard part of the roof of the mouth. The lesser palatine nerve supplies the soft palate and palatine tonsil but is not involved in supply to the hard palate

A 40-year-old woman suffers severe head trauma in a car crash. After radiographic examination she is diagnosed with a fracture of the temporal bone resulting in a lesion of the facial nerve proximal to the origin of the chorda tympani in the posterior wall of the tympanic cavity. Which of the following functions would most likely remain intact in this patient? A. Control of muscles and lower half of face B. Control of secretions by submandibular gland C. Taste sensation from anterior two thirds of tongue D. Tear production by the lacrimal gland E. Voluntary closure of the eyelid

D. The greater petrosal nerve is a branch of the facial nerve that ultimately supplies the lacrimal gland. This branch comes off the facial nerve at the geniculate ganglion proximal to the chorda tympani

A 55-year-old man is admitted to the neurosurgical clinic for a scheduled removal of a tumor in the left jugular canal. Postoperatively, the patient has no gag reflex when the ipsilateral pharyngeal wall is stimulated, although the pharynx moved upward, and a gag reflex resulted when the right pharyngeal wall was stimulated. The uvula was deviated to the right and the left vocal cord had drifted toward the midline. Which of the following structures will contain the neural cell bodies for the motor supply of the paralyzed muscles? A. Nucleus solitarius B. Trigeminal motor nucleus C. Dorsal motor nucleus D. Nucleus ambiguus E. Superior or inferior ganglia of vagus

D. The nucleus ambiguus gives rise to efferent motor fibers of the vagus nerve, which supply the laryngeal and pharyngeal muscles. If supply to this region is interrupted, an individual loses the swallowing, cough, and gag reflexes

Where is he location of the postganglionic parasympathetic neural cell bodies that directly innervate the parotid gland? A Trigeminal (semilunar Gasserian) ganglion B Inferior salivatory nucleus C. Superior cervical ganglion D. Otic ganglion E. Submandibular ganglion

D. The otic ganglion is the location of the postganglionic parasympathetic neural cell bodies innervating the parotid gland

A 45-year-old woman visits the outpatient clinic with past history of dysphagia, nighttime fits of coughing, repeated chest infections, and a palpable swelling in her neck. Radiographic examination reveals the presence of a congenital pharyngeal pouch. Between which muscles is this pouch located? A. Between styloglossus and stylopharyngeus B. Between palatoglossal arch and median glossoepiglottic fold C. Between upper and middle pharyngeal constrictors D. Between the cricopharyngeal and thyropharyngeal portions of inferior pharyngeal constrictor E. Between the middle and inferior pharyngeal constrictors

D. The pharyngeal (Zenker) diverticulum is usually located between the cricopharyngeal and thyropharyngeal portions of the inferior pharyngeal constrictor.

A 7-year-old boy with a high fever is brought to the pediatrician During physical examination the patient complains of pain in his ear. His throat appears red and inflamed, confirming the diagnosis of pharyngitis. Which of the following structures provided a pathway for the infection to spread to the tympanic cavity (middle ear)? A. Choanae B. Internal acoustic meatus C. External acoustic meatus D. Pharyngotympanic tube E. Pharyngeal recess

D. The pharyngotympanic (eustachian) tube connects the middle ear and the nasopharynx and is the conduit for spreading infections

A 5-year-old boy is admitted to the hospital with otitis media. Otoscopic examination reveals a bulging and inflamed eardrum. It is decided to incise the tympanic membrane to relieve the painful pressure and allow drainage of the infection associated with otitis media. Which of the following is the best location to make an opening (myringotomy) for drainage? A. The anterior superior quadrant of the eardrum B. The posterior superior quadrant of the eardrum C. Directly through the site of the umbo D. The posterior inferior quadrant of the eardrum E. A vertical incision should be made in the eardrum, from the 12 o'clock position of the rim of the eardrum to the 6 o'clock position of the rim.

D. The posterior inferior quadrant of the eardrum is the only portion of the tympanic membrane that would allow for an incision with minimal or no damage to adjacent important structures

A 34-year-old woman is admitted to the hospital with a large mass at her thyroid gland. Ultrasound examination reveals a benign tumor. Twenty-four hours following a partial thyroidectomy, in which the inferior thyroid artery was also ligated, the patient speaks with a hoarse voice and has difficulty in breathing on exertion. Which of the following nerves was most likely injured during the surgical procedure? A. Internal branch of superior laryngeal B. Ansa cervicalis C. Ansa subclavia D. Recurrent laryngeal E. External branch of superior laryngeal

D. The recurrent laryngeal nerve is the most likely nerve damaged during the surgery because it runs in close proximity to the inferior thyroid artery and is easily injured or transected with the artery if extreme care is not exercised during operative procedures

A 56-year-old man visits the outpatient clinic with a complaint of severe headaches and ear pain. Radiographic examination reveals a tumor in the middle ear cavity, invading through the bony floor. Which of the following structures will most likely be affected? A. The cochlea and lateral semicircular canal B. The internal carotid artery C. The sigmoid venous sinus D. The internal jugular bulb E. The aditus ad antrum of the mastoid region and the facial nerve

D. The sigmoid sinus collects venous blood from the transverse sinuses and empties it into a small cavity known as the jugular bulb, the inferior portion of which is located beneath the bony floor of the middle ear cavity.

A 55 year-old woman visits the outpatient clinic with a swelling in her neck. Radiographic and ultrasound examinations reveal a benign thyroid gland tumor. Three days after thyroidectomy the patient shows air bubbles in the CT of her brain. Which of the following is the most likely cause of the air bubbles in this case? A. Injury to inferior thyroid artery B. Injury to inferior and superior thyroid arteries C. Injury to superior thyroid artery and vein D. Injury to superior and middle thyroid veins E. Injury to superior, middle, and inferior thyroid veins

D. The superior thyroid vein is a tributary to the internal jugular vein; it accompanies the superior thyroid artery

A 5-day-old infant was born with a laryngeal defect. The greater cornu and the inferior part of the hyoid bone were absent at birth. Failure of development of which of the following embryonic structures most likely led to these defects? A. Maxillary prominence B. Mandibular prominence C. Second pharyngeal arch D. Third pharyngeal arch E. Fourth pharyngeal arch

D. The third pharyngeal arch gives rise to the greater cornu and lower part of the hyoid bone in addition to the stylopharyngeus muscle

A 56-year-old female complains of diplopia (double vision) when walking down stairs. A lesion of which of the following nerves is most likely responsible for this patient's complaint? A. Optic B. Oculomotor C. Abducens D. Trochlear E. Frontal

D. The trochlear nerve innervates the superior oblique muscle, which acts to move the pupil downward and laterally

A 70-year-old man is admitted to the hospital with severe headaches. During physical examination he has difficulty coughing and swallowing. A CT scan shows a tumor affecting a cranial nerve. Which nerve is most likely affected? A. Mandibular B. Maxillary C. Glossopharyngeal D. Vagus E. Hypoglossal

D. The vagus nerve is responsible for sensation in the mucosa of the larynx down to the level of the vocal folds, and also motor innervation of the muscles that initiate a cough reflex and swallowing (motor)

A 17-year-old female was admitted to the hospital with a high fever. Following intravenous administration of antibiotics, a routine CT scan revealed a "thoracic outlet" syndrome. Which symptom would most likely result from this syndrome? A. Problems with respiration because of pressure on the phrenic nerve B. Reduced blood flow to the thoracic wall C. Reduced venous return from the head and neck D. Numbness in the upper limb E. Distention of the internal jugular vein

D. Thoracic outlet syndrome is characterized by the presence of a cervical rib, accessory muscles, or connective tissue bands that constrict the limited dimensions of the thoracic outlet

A 7-day-old infant is admitted to the pediatric intensive care unit with microphthalmia. Which of the following is the most likely cause of this condition? A. Infection with rubella virus B. Choroid fissure failed to close C. Persistent hyaloid artery D. Toxoplasmosis infection E. Epstein-Barr virus infection

D. Toxoplasmosis infection is caused by the parasite Toxoplasma gondii , which is associated with undercooked meat and the feces of cats

A 2-day-old infant male has a noticeable gap in his upper lip. The diagnosis is a cleft lip. Failure of fusion of which of the following structures is the most likely cause of this anomaly? A. Lateral nasal and maxillary prominences/ processes B. Medial nasal prominences/processes C. Lateral nasal and medial nasal prominences/ processes D. Lateral prominences/processes E. Maxillary prominences/processes and the intermaxillary segment

E The most common cause of cleft lip is failure of fusion of the maxillary process and the intermaxillary segment.

A 72-year-old woman is admitted to the emergency department with tenderness in the upper right thorax, painful to compression. During physical examination the patient presents with slight ptosis of her right eye. The right pupil is constricted more distinctly than the contralateral pupil. Which of the following is the most likely diagnosis? A. Raynaud's disease B. Frey syndrome C. Bell palsy D. Quinsy E. Pancoast tumor

E. A Pancoast tumor is located in the pulmonary apex, usually in the right lung. These tumors can involve the sympathetic chain ganglia and cause Horner's syndrome (slight ptosis and miosis).

A 22-year-old woman visits the outpatient clinic with a painless swelling on the right side of her neck. A CT scan examination reveals a well-defined cystic mass at the angle of the mandible, just anterior to the sternocleidomastoid muscle. What is the most likely diagnosis?A. Dermoid cyst B. Inflamed lymph node C. Accessory thyroid tissue D. Thyroglossal duct cyst E. Lateral cervical cyst

E. A lateral cervical cyst is caused by remnants of the cervical sinus and would present anterior to the sternocleidomastoid

A 45-year-old female is admitted to the emergency department with visual problems when she walks down stairs. During physical examination the patient exhibits weakness of her downward medial gaze. Cerebral arteriography and CT images indicate that a nerve is being compressed by an arterial aneurysm just inferior to the tentorium cerebelli Which of the following arteries and nerves is most likely being compressed? A. Internal carotid artery/abducens nerve B. Middle cerebral artery/oculomotor nerve C Posterior cerebral artery/ophthalmic nerve D Basilar artery/ophthalmic nerve E Superior cerebellar artery/trochlear nerve

E. A lesion of the trochlear nerve causes weakness of downward medial gaze. As a result, patients with trochlear nerve lesions commonly have difficulty walking down stairs

A 49-year old woman is admitted to the hospital with headaches and dizziness. Radiographic examination reveals a tumor in the jugular canal. Upon physical examination, when the right side of the pharyngeal wall is touched with a tongue depressor, the uvula deviates to the left and the left pharyngeal wall contracts upward. When the left pharyngeal wall is touched, the response is similar. Which of the following nerves is most likely to have been injured by the tumor? A. Right glossopharyngeal B. Left glossopharyngeal C. Right mandibular D. Left hypoglossal E. Right vagus

E. A tumor of the jugular canal would likely affect the glossopharyngeal, vagus, and accessory nerves as they exit the cranium through the jugular foramen. The uvula deviates toward the unaffected side of the pharyngeal muscles because of the pull of the unopposed levator veli palatini

A 45-year-old man is admitted to the emergency department with severe dyspnea. During physical examination there is swelling in the floor of his mouth and pharynx so that his airway is nearly totally occluded. In addition, there is a swelling in his lower jaw and upper neck. His physical history indicates that one of his lower molars was extracted a week ago and he had been feeling worse every day since that event. Which of the following conditions will be the most likely diagnosis? A. Quinsy B. Torus palatinus C. Ankyloglossia D. Ranula E. Ludwig's angina

E. An infection of the submandibular space is usually the result of a dental infection in the mandibular molar area in the floor of the mouth

A 45-year-old male construction worker slips and falls on a nail protruding from a board. The nail penetrates the skin overlying the submental triangle lateral to the midline. Which of the following muscles would be the last to be penetrated? A. Platysma B. Mylohyoid C. Anterior belly of he digastric D. Geniohyoid E. Genioglossus

E. During a puncture wound as described in this case, passing up from below the chin, the nail would first pierce the platysma, then the anterior belly of the digastric, then the mylohyoid, then the geniohyoid, and finally the genioglossus

A 44-year-old patient is admitted to the hospital with Raynaud's disease. A sympathetic blocking drug is administered in high doses. Which of the following conditions will be expected to occur as an adverse effect of the drug? A. Exophthalmos and dilated pupil B. Enophthalmos and dry eye C. Dry eye and inability to accommodate for reading D. Wide-open eyelids and loss of depth perception E. Ptosis and miosis

E. Ptosis and miosis occur in response to blocking of sympathetic innervation. Ptosis (drooping of the eyelid) results from lack of innervation of the superior tarsal muscle (of Müller), and miosis (pupillary constriction) results from unopposed parasympathetic innervation of the pupil.

Early closure of the fontanelles of the infant skull can result in compression of the brain, restricting brain growth. Which of the following fontanelles is located at the junction of sagittal and coronal sutures and at what age does this fontanelle typically close? A. Posterior fontanelle, which closes at about 2 years B. Mastoid fontanelle, which closes at about 16 months C. Lambdoid fontanelle, which closes at 8 months to 1 year D. Sphenoidal fontanelle, which closes at 3 years E. Anterior fontanelle, which closes at 18 months

E. The anterior fontanelle is located at the junction of the sagittal and coronal sutures and closes at around 18 months of age

A 68-year-old man is admitted to the emergency department after an acute cerebral vascular accident (stroke). Radiographic studies reveal that the primary damage was to the anterior inferior cerebellar artery, resulting in a small hemorrhage of the artery at its origin from the main trunk. Which of the following nerves will most likely be immediately affected by the hemorrhage? A. Optic nerve B. Oculomotor nerve C. Trochlear nerve D. Trigeminal nerve E. Abducens nerve

E. The anterior inferior cerebellar artery (AICA) is a major supplier of the anterior inferior portion of the cerebellum; The abducens nerve is situated at the pontomedullary junction and is therefore most likely to be damaged following hemorrhage of the AICA

A 2-month-old male infant had a small pit at the anterior border of the sternocleidomastoid muscle, with mucus dripping intermittently from the opening. The pit extended to the tonsillar fossa as a branchial fistula. Which of the following embryologic structure(s) is (are) involved in this anomaly? A. Second pharyngeal arch B. Second pharyngeal pouch and groove C. Third pharyngeal pouch D. Thyroglossal duct E. Second pharyngeal pouch and cervical sinus

E. The child in this problem suffers from a fistula that indicates an open malformation. This implies that the defect must be due to failure of closure for both an internal and an external structure.

A 32-year-old man is admitted to the hospital with severe headache and visual problems. The dilator pupillae muscle, the smooth muscle cell fibers of the superior tarsal muscle (of Müller, part of the levator palpebrae superioris), and the smooth muscle cells of the blood vessels of the ciliary body are supplied by efferent nerve fibers. Which of the following structures contains the neural cell bodies of these fibers? A. Pterygopalatine ganglion B. IML (lateral horn) C1 to C4 C. Geniculate ganglion D. Nucleus solitarius E. Superior cervical ganglion

E. The dilator pupillae, levator palpebrae superioris, and smooth muscle cells of blood vessels in the ciliary body all receive sympathetic innervation. The postsynaptic cell bodies of the sympathetic neurons that innervate these structures are located in the superior cervical ganglion

A 60-year-old man presents with a swelling in his neck. Physical examination and biopsy show a benign tumor in his piriform recess. The mucosa of the piriform recess must be anesthetized during the removal of the tumor. Which nerve supplies general sensation to the mucous membrane of the laryngeal vestibule and piriform recesses? A. External laryngeal B. Glossopharyngeal C. Hypoglossal D. Inferior laryngeal E Internal laryngeal

E. The internal branch of the superior laryngeal nerve, often called the internal laryngeal nerve, supplies the mucosa of the larynx above the vocal folds (which includes the vestibule of the larynx) and the piriform recess. The external branch of the superior laryngeal nerve (external laryngeal nerve) is motor to the cricothyroid muscle.

A 70-year-old man is admitted to the hospital with chronic headache and enlarged lymph nodes. A CT scan shows a tumor at the jugular foramen. Which of the following would be the most likely neurologic deficit? A. Loss of tongue movements B. Loss of facial expression C. Loss of sensation from the face and the scalp D. Loss of hearing E. Loss of gag reflex

E. The jugular foramen is the route of exit for three nerves (glossopharyngeal, vagus, and accessory nerves) and one vein (internal jugular) from the cranial cavity. The glossopharyngeal nerve provides the sensory input for the gag reflex, whereas the vagus nerve provides the motor output

A 52-year-old man is admitted to the emergency department with a bullet wound in the infratemporal fossa. During physical examination it is observed that the patient has lost unilateral sensation of hot, cold, pain, and pressure from the front part of the tongue, but taste and salivary function are preserved. Which of the following is the most likely diagnosis? A. The facial nerve was transected distal to the origin of the chorda tympani. B Receptors for hot, cold, pain, and pressure are absent in the patient's tongue. C. The glossopharyngeal nerve has been injured in the pharynx. D The superior laryngeal nerve was obviously severed by the bullet. E. The lingual nerve was injured at its origin near the foramen ovale.

E. The lingual nerve supplies sensory innervation to the mucous membrane of the anterior two thirds of the tongue, taste sensation to the anterior part of the tongue, and parasympathetic fibers to the oral salivary glands.

A 40-year-old unconscious man is admitted to the emergency department after being hit in the head with a baseball. A CT scan examination reveals a fractured pterion and an epidural hematoma. Branches of which of the following arteries re most likely to be injured? A. External carotid B. Superficial temporal C. Maxillary D. Deep temporal E Middle meningeal

E. The middle meningeal artery is a branch of the maxillary artery and courses between the dura mater and skull close to the area of the pterion

A 34-year old female is admitted to the hospital because of hoarseness for the past 3 months. Radiographic examination reveals a cancerous growth in her larynx with no evidence of metastasis. In addition, the area in which the tumor is growing is characterized by very limited lymphatic drainage. Which of the following locations is most likely to contain a tumor with these characteristics? A. Anterior commissure of the vocal ligaments B. Interarytenoid fold C. Laryngeal ventricle D. Cricothyroid ligament E. Middle segment of the vocal cord

E. The middle of the vocal cord would be the most likely location of the tumor because there is no direct lymph drainage from this region. All other locations mentioned are drained by the lymphatics

A 22-year-old woman visits the outpatient clinic with a sinus infection of two weeks' duration. Physical examination reveals that the patient has focal inflammation, with mucosal edema in the inferior nasal meatus. Drainage from which of the following structures is most likely to be obstructed by this inflammation and edema? A. Anterior ethmoidal air cells B. Frontonasal duct C. Maxillary sinus D. Middle ethmoidal air cells E. Nasolacrimal duct

E. The nasolacrimal duct is the only duct that normally drains into the inferior meatus of the nose and therefore would be affected by a focal inflammation in this region.

A 22-year-old male is admitted to the emergency department and intubated. An endotracheal tube is passed through an opening between the vocal folds. What is the name of this opening? A. Piriform recess B. Vestibule C. Ventricle D. Vallecula E. Rima glottidis

E. The rima glottidis is the opening between the vocal folds and the arytenoid cartilages

A 45-year-old woman is admitted to the hospital for severe ear pain. Physical examination reveals chronic infection of the mastoid air cells (mastoiditis). The infection can erode the thin layer of the bone between the mastoid air cells and the posterior cranial fossa and spread most commonly into which of the following venous structures? A. Superior sagittal sinus B. Inferior sagittal sinus C. Straight sinus D. Cavernous sinus E. Sigmoid sinus

E. The sigmoid venous sinus empties into the internal jugular vein and drains the cranial vault. It runs along the posterior cranial fossa near the suture between the temporal and occipital bones

A 54-year-old male is diagnosed with an aneurysm of the basilar artery close to the cavernous sinus An anterior approach to the sella turcica through the nasal cavity is performed. Through which of the following routes is the surgeon most likely to enter the cranial cavity? A. Cribriform plate B. Cavernous sinus C. Frontal sinus D. Maxillary sinus E. Sphenoidal sinus

E. The sphenoidal sinus provides the most direct access to the pituitary gland, which is situated directly above this sinus.

A 70-year-old man has a biopsy of a growth on his lower lip. The biopsy reveals a squamous cell carcinoma. Which lymph nodes will most likely be first involved in the spread of the cancer cells? A. Occipital B. Parotid C. Retropharyngeal D. Jugulodigastric E. Submental

E. The submental lymph nodes drain roughly the anterior two thirds of the mouth and tongue, including the lower lips

A 43-year-old male is admitted to the hospital complaining of diplopia (double vision) when walking down stairs. During physical examination of the extraocular muscles the patient experiences diplopia, and when he is asked to turn his right eye inward toward his nose and look down, he is able to look inward but not down. Which nerve is most likely involved? A. Abducens B. Nasociliary C. Oculomotor, inferior division D. Oculomotor, superior division E. Trochlear

E. The superior oblique muscle turns the pupil downward from the adducted position. Inability to perform this motion, in conjunction with diplopia when walking down stairs, indicates damage to the trochlear nerve.

A 32-year-old woman is admitted to the hospital with headaches and dizziness. During physical examination it is noted that the patient has partial ptosis (drooping eyelid). Which of the following muscles is most likely paralyzed? A. Orbicularis oculi, lacrimal part B. Orbicularis oculi, palpebral part C. Levator palpebrae superioris D. Superior oblique E. Superior tarsal (of Müller)

E. The superior tarsal muscle (of Müller), innervated by sympathetics, assists in elevating the eyelids and holding them up. Damage would result in partial ptosis of the eyelid.

A 55-year-old woman is admitted to the hospital with difficulty swallowing. Physical examination reveals that the patient has episodes of severe headaches and frequently aspirates fluids when drinking them. A radiographic examination reveals a skull base tumor occupying the space behind the jugular foramen. Involvement of which of the following structures is most likely responsible for the findings in the patient? A. Ansa cervicalis B. Cervical sympathetic trunk C. External laryngeal nerve D. Hypoglossal nerve E. Vagus nerve

E. The vagus nerve exits the skull at the jugular foramen and is responsible for motor innervation to the smooth muscles of the trachea, bronchi, and digestive tract, in addition to the muscles of the palate, pharynx, larynx, and superior two thirds of the esophagus

A 65-year-old woman is admitted to the hospital with signs of cavernous sinus thrombosis. Radiographic examination reveals an aneurysm of the internal carotid artery within the cavernous sinus. During physical examination what sign would one first expect to see if nerve compression has occurred within the cavernous sinus? A. Inability to gaze downward and medially on the affected side B. Complete ptosis of the superior palpebra C. Bilateral loss of accommodation and loss of direct pupillary reflex D. Ipsilateral loss of the consensual corneal reflex E. Ipsilateral paralysis of abduction of the pupil

E. Within the cavernous sinus the abducens nerve is in intimate contact with the internal carotid artery. Therefore, an aneurysm of the internal carotid artery could quickly cause tension or compression on the abducens nerve.


Related study sets

Smartbook: Chapter 2 Analyzing and Recording Transactions

View Set

Personal Finance Chapter 4 & 5 Vocabulary

View Set